You are on page 1of 44

1. A patient is performing a Phase I (inpatient) cardiac rehabilitation exercise session. The physical therapist should
terminate low-level activity if which of the following changes occurs?

1. The diastolic blood pressure increases to 120 mm Hg.

2. The respiratory rate increases to 20 breaths per minute.

3. The systolic blood pressure increases by 20 mm Hg.

4. The heart rate increases by 20 bpm.

Correct Answer: 1
During Phase I (inpatient) cardiac rehabilitation, vital sign parameters with activity that warrant termination are:
diastolic blood pressure of 110 mm Hg or greater, systolic blood pressure above 210 mm Hg or an increase greater
than 20 mm Hg from resting, and a heart rate that increases beyond 20 bpm above resting. The normal resting
respiratory rate can range from 12 to 20 breaths per minute in adults, so an increase to 20 breaths per minute with
low-level activity would not be a reason to terminate the activity

2. A patient is being evaluated for possible carpal tunnel syndrome, and a nerve conduction velocity test is
performed. Which of the following findings would MOST strongly support the diagnosis?

1. Decreased latency at the elbow

2. Decreased latency at the carpal tunnel

3. Increased latency at the carpal tunnel

4. Increased latency at the forearm

Correct Answer: 3

Nerve conduction above and below the local nerve compression is usually normal. Latency is typically increased, not decreased,
across the carpal tunnel compression site. Nerve conduction above and below the local nerve compression is usually normal

3. When examining a patient with a history of alcohol abuse, a physical therapist notes that the patient demonstrates fine
resting tremors and hyperactive reflexes. The patient reports frequent right upper quadrant pain. Which of the following
additional signs is MOST likely?

1. Jaundice

2. Hyperhidrosis

3. Hypotension

4. Nocturnal cough

Correct Answer: 1
With a history of alcohol abuse and the presence of fine resting tremors and right upper quadrant pain, the patient is presenting
a history and signs and symptoms consistent with liver disease. Jaundice is a skin change associated with disease of the hepatic
system. Hyperhidrosis can be present with endocrine disorders but is not associated with liver disease. Hypotension is not listed
as a sign of liver disorders. A nocturnal cough can be associated with rheumatic fever, but is not characteristic of liver disease.

4. Which of the following sensory testing locations corresponds to the C7 nerve root?
1. Volar aspect of the little finger (5th digit)

2. Dorsal aspect of the middle finger (3rd digit)

3. Lateral aspect of the upper arm

4. Medial aspect of the upper arm

5. Which of the following examination findings would be expected in a patient who also had sustained ankle clonus?
1. An upgoing great toe when the sole of the foot is stroked

2. Weakness of ankle plantar flexors with one-repetition strength testing

3. Absence of sensation to sharp/dull testing over the posterior lower leg

4. Hyporeflexia when deep tendon reflexes are elicited in the lower leg

Correct Answer: 1
Sustained ankle clonus indicates a central nervous system dysfunction, as does the presence of a Babinski sign (that is, an
upgoing great toe with stroking of the plantar foot). The other options are associated with lower motor neuron problems.

6. A 3-month-old infant has poor midline head control. During evaluation, the physical therapist notes facial asymmetry and
observes that the infant has limitation of cervical rotation to the left and cervical lateral flexion to the right. A radiology report
indicates premature fusion of the infant’s cranial sutures. The infant MOST likely has:

1
1. right congenital muscular torticollis.

2. left congenital muscular torticollis.

3. right cervical facet hypomobility.

4. left cervical facet hypomobility.

Correct Answer: 1
The infant exhibits signs of torticollis affecting the right sternocleidomastoid muscle. Torticollis is named for the side of the
limited lateral flexion. Asymmetry and premature closure of sutures (plagiocephaly) are not typically seen with cervical facet
hypomobility in infants.

7. A patient had a split-thickness skin graft for a partial-thickness burn injury to the upper extremity. The surgeon has requested
range-of-motion exercises for the patient. Currently, the patient is able to actively move the upper extremity through one-third
of the range of motion for shoulder flexion. Based on this finding, what is the MOST appropriate action for the physical therapist
to take at this time?
1. Defer any range-of-motion exercises until the patient is able to participate more actively.

2. Begin active assistive range-of-motion exercises.

3. Begin bed mobility training to facilitate increased use of the upper extremity.

4. Continue with active range-of-motion exercises.

Correct answer: 2
Deferring any range-of-motion exercises is not a practical choice, as contracture will develop postoperatively. Because this
patient cannot achieve full range of motion by himself, active assistive range of motion is indicated to prevent contracture
postoperatively. Although bed mobility training is a creative way to possibly increase upper extremity range of motion, given the
acuity of the patient’s surgical wound, the patient would need more range of motion for this intervention to be more beneficial.
Continuing with only active range of motion would not facilitate adequate increases in range of motion and would not prevent
contractures.

8. Which of the following modalities BEST addresses the cause of calcific tendinitis in the bicipital tendon?
1. Sensory level interferential current at 80 Hz to 100 Hz

2. Iontophoresis with acetic acid at 60 mA/minute

3. High-volt pulsed electrical stimulation at 200 pps

4. Diathermy with a parallel treatment set-up

Correct answer: 2
Sensory level interferential current at 80 to 100 Hz does not address the problem itself but may address any related pain.
Iontophoresis with acetic acid can address the cause of calcific tendinitis, not just the symptoms. High-volt pulsed electrical
stimulation at 200 pps does not address the problem itself but may address any related pain. Diathermy with a parallel
treatment set-up is not the best choice, as deep heat will not address the pathology.

9. To minimize skin irritation during functional electrical nerve stimulation, a physical therapist should use:
1. lower intensity, larger interelectrode distance, and larger electrodes.

2. lower intensity, larger interelectrode distance, and smaller electrodes.

3. higher intensity, smaller interelectrode distance, and smaller electrodes.

4. lower intensity, smaller interelectrode distance, and larger electrodes.

Correct answer: 1
Several things can be done to decrease the current density and the possibility of skin irritation.
These include decreasing the intensity of the stimulation, increasing the inter-electrode distance, and using larger electrodes.

10. Manual muscle testing of a patient’s pelvic floor muscles reveals a grade of Poor (2/5). Which of the following positions is
BEST to begin strengthening?
1. Supine

2. Standing

3. Seated

4. Walking

Correct answer: 1
A grade of Poor (2/5) is defined as full excursion in a gravity-eliminated position. All of the other options are against gravity
positions,
which would be inappropriate, given this grade of weakness.

11. During an examination of elbow strength using manual muscle testing, a patient supinates the forearm when attempting
elbow flexion. Which of the following muscles is MOST likely doing the major part of the work?
1. Biceps brachii

2. Brachialis

3. Supinator

2
4. Brachioradialis

Correct answer: 1
The biceps brachii is both an elbow flexor and supinator, and it is most effective as a supinator with the elbow flexed to about
90°
(approximately the muscle testing position). The brachialis does not cause supination (only flexion).
The supinator does not flex the elbow. The brachioradialis would move the forearm to a midposition rather than fully supinating
it.
Therefore, when the elbow both flexes and supinates, the biceps brachii would be the most likely muscle causing this action.

12. During evaluation of a patient’s balance, a physical therapist gently pushes the patient backward slightly and observes how
the patient recovers from the perturbation. What strategy is the patient MOST likely to use to correct for the perturbation?
1. Knee

2. Hip

3. Ankle

4. Stepping

Correct answer: 3
Regarding options 1 and 2, for larger perturbations, individuals utilize hip and knee muscles to recover the balance.
Regarding option 3, for slight perturbations, most individuals use an ankle strategy. Ankle musculature is used to control the
perturbation and recover the balance. Regarding option 4, if the perturbation is strong enough to cause the individual’s
center of mass to move outside the base of support, a stepping strategy would be employed by taking a step and
increasing the size of the base of support.

13. Which of the following findings BEST describes normal capillary filling?
1. Rebound vasodilation after icing

2. pressure of 120/76 mm Hg

3. Pulse oximetry measurement of 98%

4. Blanching of the nail bed with color return in <3 seconds

Correct answer: 4
By definition, the blanching of nail bed with color return in < 3 seconds is normal capillary refill.

14. Which of the following instructions is MOST appropriate for teaching a patient with C6 quadriplegia to transfer from a
wheelchair to a mat?
1
Keep fingers extended to give a broader base of support.
.

2
Rotate head and shoulders in the same direction as the desired hip motion.
.

3
Rotate head and shoulders in the direction opposite to the desired hip motion.
.

4
Keep both hands next to the knees to lock the elbows.
.

Correct Answer: 3
The position described creates the necessary force to move the lower body in this transfer, given the level of the spinal cord
injury.
Finger extension against resistance would be difficult for a patient with C6 quadriplegia.
The patient’s hands would be kept near the thigh or hips with one hand on the mat and one on the wheelchair.

15. A patient has a spinal cord injury that resulted in damage to the sacral segments and disruption of the sacral reflex arc. The
patient is MOST likely to have which of the following characteristics?
1.
Voluntary control of defecation

2
Tonic contraction of the external anal sphincter
.

3
Flaccidity of pelvic floor musculature
.

4
Permanent absence of the gastrocolic reflex
.

Correct answer: 3
The external anal sphincter and pelvic floor muscles are composed of striated muscle fibers. They receive somatic innervation
from sacral cord segments 2 through 4. With damage to these segments, the sphincter and the pelvic floor muscles remain
flaccid. The individual loses voluntary control of defecation. The gastrocolic reflex, mediated by the intrinsic nervous system of
the GI tract, returns after resolution of spinal shock.

3
16. A patient who reports double vision has ptosis, lateral strabismus, and a dilated pupil in the left eye. Which of the following
cranial nerve test results is MOST likely to be abnormal?
1. Pupillary light reflex

2. Facial muscle strength

3. Jaw-jerk reflex

4. Pain sensation on the face

Correct answer: 1
The cranial nerve involved is the oculomotor nerve. This nerve innervates the medial rectus, which, if weak, would cause a
lateral strabismus. The oculomotor nerve is also responsible for mediating papillary constriction and a lesion would cause
papillary dilation. The ptosis is caused by loss of innervation to the levator palpabrae superioris muscle, which elevates the
eyelid. The double vision would be caused by the inability to move the eyeball normally, because four of the six ocular muscles
are controlled by the oculomotor nerve. The oculomotor nerve is also important in mediating the pupillary light reflex. The facial
nerve innervates the muscles of facial expression. The trigeminal nerve mediates the jaw-jerk reflex and pain sensation from the
face.

17. In addition to standard precautions, what other precaution should a physical therapist observe when working with a patient
infected with methicillin-resistant Staphylococcus aureus?
1. Airborne

2. Sterile

3. Droplet

4. Contact

Correct answer: 4
Since methicillin-resistant Staphylococcus aureus is spread by contact, wearing a face shield or mask is not necessary. Sterile
precautions or techniques are not necessary for the physical therapist to use with a patient infected with methicillin-resistant
Staphylococcus aureus.

18. The physical therapist is positioning a patient for postural drainage. To BEST drain the posterior segment of both lower
lobes, the patient should be placed in which of the following positions?
1.
Prone, head down at a 45° angle

2.
Supine, flat surface

3.
Sidelying, head elevated at a 30° angle

4.
Sitting, leaning forward

Correct answer: 1
The best position for draining the posterior segment of both lower lobes would be prone lying with the head down and the lower
extremities and hips elevated to about 45°.

19. Clubbing of the fingers is MOST associated with which of the following pathologies?
1.
Lymphedema

2.
Pulmonary disease

3.
Chronic venous insufficiency

4. Complex regional pain syndrome

Correct answer: 2
Conditions that chronically interfere with tissue perfusion and nutrition may cause clubbing. Pulmonary disease is the most
predominant cause of digital clubbing, present 75% to 85% of the time clubbing is noted. Since the other conditions also affect
tissue perfusion, they could contribute to the condition, but are not listed as common findings.

20. A patient who is 8 months pregnant has an abdominal diastasis recti with a separation of 1.5 in (4 cm). Which of the
following exercises would be the MOST appropriate initial exercise for abdominal strengthening in a supine position?
1.
Trunk curls

2.
Hooklying head lifts

3.
Pelvic-tilt leg sliding

4.
Bilateral leg lowering

4
Correct answer: 2
Trunk curls are contraindicated for a patient with diastasis recti.
Supine hooklying head lifts emphasize the rectus abdominis muscle and are least likely to increase the separation of the
diastasis recti.Pelvic-tilt leg sliding is more advanced than head lifts. Bilateral leg-lowering is an advanced abdominal
strengthening
exercise that causes excessive low back strain and should not be performed during pregnancy.

21. A patient with Parkinson disease has just been admitted to a rehabilitation unit. The patient is dependent in all transfers and
requires moderate assistance of one person to walk 30 ft (9.1 m) with a standard walker. To facilitate good carryover for
activities, instruction of the family in transfers should occur:
1.
during a home visit after the patient is discharged.

2.
just prior to discharging the patient.

3.
early in the rehabilitation program.

4.
when the family feels ready to take the patient home.

Correct answer: 3
The family should be involved in all stages of planning and treatment. Family involvement can shorten the rehabilitation process
and facilitate the patient’s return to the community. It is important to have the family involved early in the rehabilitation process
rather than wait until the patient is ready to be discharged.

22. A patient with a right transfemoral prosthesis will be able to maintain the knee in extension while weight bearing if the
center of gravity of the body is shifted so that the gravitational line falls:
1.
posterior to the axis of the right knee joint.

2.
lateral to the axis of the right knee joint.

3.
anterior to the axis of the right knee joint.

4.
medial to the axis of the right knee joint.

Correct answer: 3
Static alignment for knee stability is established by positioning of the knee so that the lateral reference line falls anterior
to the knee joint.

23. During manual muscle testing of the hip flexors in the sitting position, a patient exhibits lateral (external) rotation with
abduction of the thigh as resistance is applied. The physical therapist should suspect muscle substitution by the:
1.
sartorius.

2.
tensor fasciae latae.

3.
adductor longus.

4.
semimembranosus.

Correct answer: 1
The sartorius flexes, laterally (externally) rotates, and abducts the hip joint. With resisted hip flexion, the sartorius will be
recruited
to perform all three actions, giving the observed substitution pattern. The tensor fasciae latae is a medial (internal) rotator and
flexor of the hip, so substitution by it would involve medial (internal) rotation and abduction. The adductor longus would adduct
the hip. Substitution by the semimembranosus would cause hip extension.

24. A physical therapist reads that the interrater reliability of a new hand-held dynamometer is .93. What is the MOST
appropriate interpretation of this value?
1.
Similar scores were obtained for a group of subjects when different therapists measured the subjects.

2.
Similar scores were obtained for a group of subjects when the same therapist repeated the measures.

3.
Dissimilar scores were obtained for a group of subjects when different therapists measured the subjects.

4.
Dissimilar scores were obtained for a group of subjects when the same therapist repeated the measures.

Correct answer: 1
Interrater reliability concerns variation between two or more raters who measure the same group of subjects.
The reliability coefficient has values from 0.00 to 1.00. A reliability of 1.00 means there was total agreement.
Thus a value of .93 means there was a high degree of agreement on the scores from the dynamometer among several
therapists.
Agreement of measures by the same therapist would be intrarater reliability.

25. A physical therapist researcher is developing a study to compare differences in range of motion outcomes in

5
two groups of patients who have had a total knee arthroplasty. Over an 8-week period, one group of patients receives
outpatient physical therapy 1 time/week and the other group receives outpatient physical therapy 3 times/week. In this study,
what is the dependent variable?

1.
Range of motion

2.
Frequency of visits

3.
Total knee arthroplasty

4.
8 weeks

Correct answer: 1
The dependent variable (ROM) is the factor that is caused by the independent variable (frequency of visits).

26. Which of the following teaching strategies is MOST appropriate for an older adult patient with mild dementia?
1.
Use auditory or visual input separately.

2.
Use metaphors to reinforce new concepts.

3.
Establish a consistent pace for teaching.

4.
Present one piece of new information at a time.

Correct Answer: 4
To minimize confusion, the patient should be presented with one new item at a time. The pace of learning should be set by the
patient. Visual and auditory input together can promote learning. Concrete examples are easier to understand than metaphors.

27. A patient with chronic venous insufficiency of the lower extremities is MOST likely to exhibit:

1. normal superficial veins, no edema, ulceration, and patches of gangrene around the toes.

2. dilation of superficial veins, edema, and stasis ulceration.

3. no edema, faint dorsalis pedis pulse, and cold, hairless extremities.

4. dilation of superficial veins and edema made worse during sitting or elevation of the lower extremities.

Correct answer: 2
With venous insufficiency, the limbs would be edematous and the superficial veins would be dilated. If the venous insufficiency is
not corrected, ulceration could develop. Options 1 and 3 are ruled out because they indicate no edema. Option 4 is not correct
because
the condition is relieved by sitting or leg elevation.

28. A patient with muscular dystrophy was removed from mechanical ventilation 1 day ago. The patient is currently unable to
independently clear secretions, despite receiving instruction in the bronchopulmonary hygiene techniques of positioning,
percussion, shaking, and vibration. Which of the following interventions is MOST appropriate to help this patient clear the
secretions?

1. Nasotracheal suctioning

2. Manual costophrenic assist

3. Supplemental oxygen

4. Inspiratory muscle training

Correct answer: 2
While nasotracheal suctioning is a viable option to clear a patient’s secretions, it is usually the last resort when a patient does
not
have an artificial airway. So if a manual costophrenic assist doesn’t work, then suctioning may be needed. Any patient who has
been
receiving mechanical ventilation will likely have some respiratory muscle deconditioning. However, with superimposed
neuromuscular
disease, the respiratory muscle weakness will be further exacerbated. Therefore, providing manual assist at the lower ribs
during
cough will assist the patient in successfully clearing secretions. Supplemental oxygen can help a patient’s ventilatory
muscle endurance but does not ensure that the patient will be able to generate enough force during the cough.
Inspiratory muscle training would be beneficial for this patient, but not at this acute stage just after removal of ventilation.
Once secretions are under control, then muscle training can begin.

29. An older adult patient had a total hip arthroplasty following a hip fracture from a fall in his home. The patient lived alone and
was previously able to perform all activities of daily living independently. He has been admitted to a skilled nursing facility for
rehabilitation. The patient’s goal is to return home. His family believes he should be admitted to a long-term care facility for his
own safety after acute rehabilitation is completed. The physical therapist’s MOST appropriate action is to:
1. advise the family that the patient should determine his own discharge environment.
6
1.
advise the family that the patient should determine his own discharge environment.

2.
recommend a team conference with the patient and his family to discuss discharge plans.

3.
schedule a home visit to determine if home modifications are needed before discharge.

4.
implement a treatment plan with a long-term goal of discharge to home.

Correct answer: 2
The best answer would be for the therapist to recommend a team conference. That conference would bring together members of
the
health care team, as well as the patient and family members, to arrive at joint decision for placement of the patient. The best
time
for such a meeting would be prior to discharge, because such meetings are difficult to arrange after discharge. Allowing the
patient
to determine his own discharge environment may be fine if there are no safety concerns; however, a decision whether or not the
patient is competent to make that decision must be made first. Scheduling a home visit or the implementation of a treatment
plan
for home care would be premature until a decision on placement has been made.

30. Which of the following reasons is the PRIMARY purpose for applying statistical analyses to single-subject research?
1.
To substantiate visual analysis of graphically displayed data

2.
To determine whether observed changes are real or chance occurrences

3.
To advocate changes in intervention methods

4.
To establish credibility for the intervention technique

Correct answer: 2
Data analysis in single-subject research is based on evaluation of measurements within and across design phases, to determine
if
behaviors are changing and if observed changes during intervention are associated with the onset of treatment.
While visual analysis of graphic display of data is the most commonly used data analyses method in single-subject design,
statistical analysis provides a more quantitative approach to determine whether observed changes are real or chance
occurrences.

31. Upon removal of a wet-to-dry dressing from a patient’s draining wound, a physical therapist observes that the skin
immediately surrounding the wound is macerated. What should the therapist recommend for future wound care?
1.
Continue using the current dressing type.

2.
Make the dressing more absorbent.

3.
Leave the dressing on longer between dressing changes.

4. Change to a pressure-type dressing.

Correct answer: 2
Macerated tissue results from excessive moisture. A more absorbent dressing would soak up the excessive moisture and prevent
the maceration.

32. A 50-year-old patient had an uncomplicated open repair of a rotator cuff tear 2 weeks ago. The patient asks the physical
therapist when the shoulder will be normal again. Which of the following expected outcome time frames MOST accurately
addresses this patient’s question?
1.
3 weeks to lift a 5-lb (2.3-kg) object

2.
3 weeks to sleep on the involved side

3.
3 months to lift the upper extremity overhead to reach into a cabinet

4.
3 months to play golf

Correct answer: 3 Three weeks is too early to lift 5-lb (2.3-kg) objects. Three weeks is too early to sleep on the involved
side. Usually by 8 to 12 weeks, a patient who has had an uncomplicated open repair of a rotator cuff 2 weeks ago is able to
actively elevate the arm to functional heights. Three months is too early to play golf.

33. A physical therapist is preparing to evaluate a patient who had a closed reduction with cast fixation for an ankle fracture 1
day ago and currently has non-weight-bearing status. The patient reports foot pain in the injured extremity. Based on the
patient’s report, which of the following procedures should the physical therapist perform?
1. Gait examination

7
1.
Gait examination

2.
Transfer abilities

3.
Capillary refill in toes

4.
Blood pressure examination

Correct answer: 3
Since the patient is non-weight-bearing, transfer abilities and gait examination are less likely to yield information regarding the 

cause of foot pain. With regard to capillary refill in toes, complications of cast fixation may include swelling and, if severe,
compartment syndrome. Improperly fitted casts and/or patients leaving limbs in a dependent position can result in painful
swelling
that creates occlusion to distal blood flow. Testing capillary refill is an easy test to examine distal extremity perfusion.
Blood pressure changes could cause changes in perfusion to distal extremities, but it would be very difficult to measure
lower extremity blood pressure with the cast on.

34. Which of the following motions are MOST restricted with a hip capsular pattern?
1.
Medial (internal) rotation and abduction

2.
Lateral (external) rotation and abduction

3.
Extension and adduction

4.
Flexion and lateral (external) rotation

Correct answer: 1
The capsular pattern of restriction at the hip is loss of medial (internal) rotation and abduction, followed by a loss of flexion and
extension; loss of lateral (external) rotation is insignificant.

35. The parent of a 4-year-old child who has myelomeningocele is interested in obtaining orthoses for the child’s gait training.
The child has an L1 neurological level lesion. Which of the following orthoses is the MOST appropriate selection for the physical
therapist to discuss with the parent?
1. Hip-knee-ankle-foot with locked hips

2. Reciprocating-gait

3. Knee-ankle-foot with a pelvic band

4. Ankle-foot

Correct answer: 2
Options 1, 3, and 4 would provide insufficient orthotic support for the child to walk. A child with an L1 lesion can walk only with
support of reciprocating gait orthoses or thoracic lumbar sacral orthosis.

36. A patient who has rheumatoid arthritis comes to physical therapy with signs of muscle atrophy, ecchymosis, puffy cheeks,
and a diagnosis of osteoporosis. Which of the following medications is the patient MOST likely receiving?
1. Penicillin (Ampicillin)

2. Prednisone (Deltasone)

3. Acetylsalicylic acid (aspirin)

4. Gold salts

Correct answer: 2
Prednisone is a glucocorticoid that exhibits the side effects described in the stem. The primary side effect of penicillin is an
allergic reaction, such as skin rashes and difficulty breathing. The primary side effect of aspirin is gastritis. The primary side
effect of gold therapy is diarrhea, irritation of oral mucosa, and skin rashes.

37. Regular aerobic exercise lasting at least 30 minutes should be MOST beneficial in decreasing the pathology associated with
which of the following conditions?
1.
Type 1 diabetes

2.
Type 2 diabetes

3.
Multiple sclerosis

4.
Amyotrophic lateral sclerosis

Correct answer: 2
Exercise is a major contributor in controlling hyperglycemia in type 2 diabetes by improving skeletal muscle glucose transport
and

8
whole-body glucose homeostasis. Regular exercise can help the body respond to insulin and is known to be effective in
managing
blood glucose. Exercise can lower blood glucose and possibly reduce the amount of medication needed to treat diabetes, or even
eliminate the need for medication. Exercise has not been shown to improve glycemic control for the person with type 1 diabetes.
Patients with degenerative neuromuscular diseases such as multiple sclerosis should use caution when exercising to avoid
excessive
fatigue, which can result in permanent losses in strength. Respiratory system impairment in individuals with multiple sclerosis
and amyotrophic lateral sclerosis may lead to poor tolerance of intense aerobic exercise.

38. A patient has been receiving physical therapy for a large wound on the lower leg. After 3 weeks of treatment, the wound is
free of necrotic tissue, but a copious amount of exudate is present. Which of the following dressings is MOST appropriate for the
wound at this time?
1.
Saline-saturated gauze

2.
Semipermeable film

3.
Gauze impregnated with zinc oxide

4. Hydrocolloid paste

Correct answer: 4
Saline-saturated gauze is not advised to control copious amounts of drainage. Semipermeable film cannot absorb copious
amounts
of exudate. Gauze impregnated with zinc oxide has not been shown to be beneficial. Hydrocolloid paste is the only option given
that
would be appropriate for managing wounds with high levels of exudate.

39. A patient with a right transtibial prosthesis describes right-knee buckling in the foot flat (loading response) stance phase of
gait. Which of the following conditions in the patient is MOST likely present to cause this problem?
1. Excessive plantar flexion

2. Stiff heel cushion

3. Low shoe heel

4. Excessive foot inset

Correct answer: 2
Excessive prosthetic plantar flexion can cause insufficient knee flexion. A stiff heel cushion can cause excessive knee flexion and
therefore buckling. A low prosthetic shoe heel can cause delayed knee flexion. Excessive prosthetic foot inset can cause
excessive
lateral thrust.

40. Which of the following recommendations is MOST appropriate for a patient with gastroesophageal reflux disease?
1.
Sit upright for at least 10 minutes after a meal.

2.
Lie on the right side before performing exercises.

3.
Head-lifting exercises should be performed in the upright position.

4.
Activities that require the supine position should be performed before a meal.

Correct answer: 4
Exercises that require a supine position should be performed before eating so that the stomach is relatively empty.
The recommended time to remain upright after a meal is 3 hours. Head lifting in supine is the recommended exercise to
strengthen the upper esophageal sphincter muscle. If performed in upright, the head falls into gravity and the exercise is no
longer
resistive. A patient should lie on the left side to reduce reflux, because lying on the right side makes it easier for acid to flow into
the esophagus, since the lower esophagus bends to the left and this straightens out with right sidelying.

41. A physical therapist is completing an examination of an inpatient with multiple comorbidities who had a total knee
arthroplasty 2 days ago. The therapist observes a bluish discoloration of the toes on the operative extremity. This finding should
be documented as:
1. cyanosis.

2. deep vein thrombosis.

3. Raynaud disease.

4. cardiac pathology.

Correct answer: 1
Cyanosis is described in the stem. Although this finding often accompanies cardiac / pulmonary pathology or may occur with
hematological or central nervous system disorders, the objective description is cyanosis.

42. A physical therapist is evaluating a 70-year-old female patient who reports the onset of midthoracic pain after working in a
garden for several hours. The presence of which of the following history items should increase the therapist’s suspicion of a
thoracic compression fracture in the patient?
1. Bowel and bladder dysfunction
9
1. Bowel and bladder dysfunction

2.
Smoking and prolonged steroid use

3.
Hypertension and diabetes

4.
Emphysema and hormone replacement therapy

Correct answer: 2
Bowel and bladder dysfunction are related to spinal cord/upper motor neuron dysfunction. Smoking and steroids can weaken
bone
and increase the likelihood for a compression fracture. Hypertension and diabetes are not risk factors for a compression fracture.
By themselves, neither emphysema nor hormone replacement therapy increases the likelihood of a compression fracture.

43. Which of the following modalities is MOST appropriate for decreasing pain and increasing tissue extensibility prior to active
hand exercises in a patient with rheumatoid arthritis?
1.
Cold gel pack

2.
Direct contact ultrasound

3.
Continuous short-wave diathermy

4.
Paraffin wax bath

Correct answer: 4
Paraffin wax baths are used in the nonflare phases to decrease pain and increase tissue extensibility in patients with rheumatoid
arthritis. The increase in collagen extensibility associated with heating may decrease pain perception and increase tolerance for
and participation in active exercise. Option 1 is incorrect because cyrotherapy (cold gel pack) decreases tissue extensibility.
Options 2 and 3 are incorrect because these modalities are not easily applied over areas with thin soft tissue (hands and
fingers).
Paraffin allows for even distribution of heat to fingers (all joints).

44. Which of the following trunk strengthening exercises is BEST for a patient to help minimize the complications associated
with osteoporosis?
1. Flexion

2. Rotation

3. Extension

4. Lateral flexion

Correct answer: 3
Trunk extension is the safest exercise for patients with osteoporosis and is necessary, given the high risk for kyphosis with
vertebral fractures. The other trunk motions are contraindicated for patients with osteoporosis.

45. A physical therapist is working on progressive functional mobility with a patient who had a transverse colectomy 2 days ago.
The patient has developed a low grade fever. What is the MOST appropriate examination for the therapist to perform prior to
continuation of the patient’s intervention?
1.
Heart rate

2.
Blood pressure

3.
Respiratory rate

4.
Auscultation

Correct answer: 4
Changes in heart rate, blood pressure, or respiratory rate can accompany fever, but none of those measurements will help in
delineating the cause of the fever. Common causes of immediate postoperative fevers are atelectasis or pneumonia. Ausculation
of
the lungs would help the therapist in delineating this as a cause of the fever and determining the
appropriate physical therapy intervention.

46. A patient is entering a cardiac rehabilitation program. The physical therapist should FIRST ask the patient to:
1. describe the correct aspects of exercise demonstrated by the therapist.

2. list problems associated with poor nutritional habits.

3. identify the harmful effects of smoking with regard to cardiac disease.

4. describe the type of angina that the patient experiences.

10
Correct answer: 4
In order to best intervene with a patient who has had cardiac dysfunction, a full examination and evaluation are necessary to
properly form a treatment plan. An important aspect of the examination is ascertaining the type of angina that the patient
experiences so that the therapist will know how to prevent angina with exercise or recognize it if it does occur during the
treatment
session. The other options provided are all outcomes that would occur after the patient has completed a cardiac rehabilitation
program.

47. A physical therapist who works in a home health agency is treating a patient with diabetes mellitus. The patient reports
that he is longer taking insulin. The therapist’s FIRST course of action should be to:

1. instruct the patient in the proper technique for injection of insulin.

2. contact the patient’s home health nurse.

3. tell the patient’s family to report this information to the physician.

4. have the patient perform a urine glucose test while the therapist is in the home.

Correct answer: 2
Contacting the patient’s home health nurse is the appropriate first course of action because of the possible safety risk associated
with the patient’s not taking insulin. Injections and glucose testing would go beyond the scope of practice for physical therapy.
Placing the responsibility on the family would not be appropriate because of the gravity of the situation.

48. A measurable objective for a community education program on proper exercise techniques would be for participants to:
1. understand the importance of a sufficient warm-up period.

2. list five stretching techniques that can be used when warming up.

3. overcome lower extremity problems and adhere to a regular exercise program.

4. appreciate the effects of increasing intensity of exercise on heart rate.

Correct answer: 2
Objectives must be measurable, and the specific behavior expected should be stated. Understanding, overcoming,
and appreciating elude tangible measurement, while listing five techniques is an activity that can be documented and is
therefore measurable.

49. While ascending stairs, an older adult patient leans forward with increased hip flexion. Which of the following muscles is
being used to BEST advantage with this forward posture?

1. Rectus femoris

2. Tensor fasciae latae

3. Semitendinosus

4. Lumbar paraspinal

Correct answer: 3
The hamstrings are hip extensors that are more active when the hip is in flexion, especially in functional activities such as stair
climbing. In this case, the patient flexes the hip, placing the semitendinosus on stretch and increasing its moment arm and
increasing its ability to produce hip extension. Therefore, the patient is leaning forward to maximize the ability of the hamstrings
to extend the hip during the stair climbing activity. The other muscles listed would not benefit as much from the increased hip
flexion.

50. The authors of a research article describe a favorable study outcome as insignificant because the study results were not
statistically significant. The insignificant finding would MOST likely be due to poor research methods if which of the following
study characteristics was present?

1. Small sample size

2. Small within-group variance

3. 85% analysis power

4. Controlled alpha level

Correct answer: 1
A small sample size can be linked to errors in statistical conclusions because there may not be enough participants to allow
application of the findings to all patients in the population of interest. The within-group variance being smaller enables less
overlapping of sample distributions. Greater than 80% analysis power is linked with a higher probability that study conclusions
are accurate. An inflated alpha level (i.e., noncontrolled) results in a higher level of statistical significance and greater
probability of a type 1 error.

51. As a patient progresses in learning a new motor skill, there should be a resultant increase in which of the following types
of feedback?

1. Tactile

2. Visual

11
3. Intrinsic

4. Verbal

Correct answer: 3
As learning progresses, feedback should progress from extrinsic (which included tactile, visual, verbal) to intrinsic feedback from
the muscle spindle and joint receptors.

52. A patient is undergoing a treadmill stress test. The appearance of abnormally wide, irregularly spaced QRS complexes on
the electrocardiogram represents:

1. ventricular depolarization.

2. premature ventricular contractions.

3. atrial fibrillation.

4. atrial repolarization.

Correct answer: 2
The QRS complex represents ventricular contraction. Atrial contraction is represented by the P wave. When an area of the
ventricle
becomes irritable and develops an ectopic foci, the ventricle will depolarize prematurely before the normal conduction sequence
(e.g., prior to SA node firing in the atria) and presents as a wide, irregularly spaced QRS complex. In normal ventricular
depolarization, the QRS would be narrow and regularly spaced and atrial fibrillation would appear as the P wave changes.
Atrial repolarization occurs within the QRS complex and is not normally visible on EKG.

53. While working in a private practice clinic, a physical therapist observes a patient fall in the parking lot outside the office.
The patient sustains a severe laceration to the forearm. The therapist secures a pressure dressing to the wound site but notes
that blood is soaking through the dressing and the bandage. Which of the following actions should the therapist perform
NEXT?

1. Elevate the limb and apply pressure to the wound.

2. Remove the dressing and bandage and start over with a tighter bandage.

3. Apply additional dressings and bandages and apply pressure to the brachial artery.

4. Call the patient’s physician and arrange transportation for medical care.

Correct answer: 1
Elevation and pressure is the correct next step in controlling bleeding. Removal of the dressing and bandage would potentially
irritate the wound and increase the hemorrhage. Elevation and pressure should be applied first, before additional dressings,
bandages,
and pressure to the brachial artery. While calling the patient’s physician to arrange transportation for medical care may be
necessary, it does not immediately control the bleeding.

54. A physical therapist is conducting a reflex text as shown in the photographs. The arrow indicates the path of the applied
stimulus. The results of the test are MOST likely to indicate:

1. a peripheral nerve injury.

2. a lesion of the anterior horn cells.

3. a normal response.

4. an injury to the spinal cord.

Correct answer: 1
The therapist is testing for a positive Babinski sign. When the Babinski sign is positive, the toes flare at the end of the test.
The presence of a positive Babinski sign indicates an upper motor neuron lesion such as a spinal cord injury. If no upper motor
neuron lesion is present, the toes will flex. The only upper motor neuron lesion is injury to the spinal cord. Peripheral nerve
injuries and lesions of the anterior horn cells are lower motor neuron lesions.

55. The work modification (standing) shown in the photograph is MOST appropriate for a patient with which of the following
pathologies?

1. Central lumbar stenosis

2. Deep vein thrombosis

3. Thoracolumbar scoliosis

4. Posterolateral lumbar disc bulge

Correct answer: 4
The photograph shows a standing desk/work station. A patient with stenosis will do better in sitting, not standing. A patient
with a deep vein thrombosis will need to move, not be stationary. A scoliosis alone does not warrant a standing work station.
Sitting increases intradiscal pressure, so standing is often preferred to sitting.

12
56. A long-distance runner comes to physical therapy with insidious onset of lower leg pain. The patient’s examination results
reveal weakness of toe flexion and ankle inversion. The physical therapist suspects vascular compromise associated with this
presentation. Palpation at which of the following locations is MOST likely to reveal diminished arterial pulses in the patient?
1.
Dorsal aspect of the foot

2.
Posterior to the lateral malleolus

3.
Posterior to the medial malleolus

4.
Lateral aspect of the popliteal fossa

Correct answer: 3
The posterior tibial artery is most likely to be involved in chronic posterior compartment syndrome, which is described in the
question. This artery should be palpated posterior to the medial malleolus. The dorsal aspect of the foot is the site for palpation
of the dorsalis pedis pulse. The dorsalis pedis artery is not involved in posterior compartment syndrome. The palpation site
posterior to the lateral malleolus is used to access the fibular (peroneal) artery. The fibular (peroneal) artery is not involved in
posterior compartment syndrome. Palpation in the popliteal fossa is a poor choice of palpation location in association with
posterior compartment syndrome. The popliteal artery may be palpated here, but the compartment syndrome manifestations
are
distal to this site.

57. During an intervention session with a patient with a recent onset of hemiplegia, a physical therapist plans to focus on
transfers to and from a wheelchair, a bathtub, and an automobile. Which of the following methods should be MOST effective for
long-term retention of these skills?
1.
Practice the activities in random order.

2.
Sequence the activities from easiest to most difficult.

3.
Allow the patient to determine the sequence of activities.

4.
Establish a predictable but variable practice sequence.

Correct answer: 1
It has been determined that a critical factor in improving learning is that the subject must do something different on consecutive
trials. Therefore the traditional approach for retraining by practicing one skill repeatedly is not the most effective. It is believed
that having a patient practice a number of tasks in random order would probably be more successful for long-term retention.
The other options do not provide for randomization of the activities and are therefore less likely to provide for long-term
retention.

58. A patient is lying supine with hips and knees extended and hands behind the head. The patient is able to raise the head,
shoulders, and thorax from the treatment table but is unable to come to a complete long-sitting position. What muscle should
the physical therapist target for a strengthening program?

1. Iliopsoas

2. External abdominal oblique

3. Quadratus lumborum

4. Upper rectus abdominis

Correct answer: 1
The abdominal muscles are active during a sit-up (with the knees extended) up until the spine is completely flexed (head,
shoulders,
thorax lifted from surface). In order to come to a long-sitting position, however, the hips must be flexed, and the abdominals
cannot perform this action because they do not cross the hip joint. Therefore, the hip flexors (iliopsoas among others) would
have to complete this motion. The inability to achieve a long-sitting position would suggest weakness in the iliopsoas muscle.

59. A patient has medication-induced Cushing syndrome. Which of the following physiological problems is a common
manifestation of this disorder?

1. Hypotension

2. Hypercalemia

3. Muscle catabolism

4. Decreased protein metabolism

Correct answer: 3
Corticosteroid-induced myopathies are common after prolonged use of the drug, and they are common in the proximal
musculature
of the extremities. The other options are the opposite of symptoms commonly found with steroid-induced conditions. Usually
there
are problems with hypertension, hypocalemia, and increased protein metabolism.

13
60. A 90-year-old patient with chronic congestive heart failure has been nonambulatory and has resided in a nursing home for
the past year. The patient was recently admitted to the hospital after an episode of dehydration. Which of the following plans
for prophylactic respiratory care is MOST appropriate?

1. Turning, coughing, and deep breathing every 1 to 2 waking hours

2. Vigorous percussion and vibration 4 times/day

3. Gentle vibration with the foot of the bed elevated 1 time/day

4. Segmental postural drainage using standard positions throughout the day

Correct answer: 1
A patient who is immobile and restricted to bed is at risk for developing atelectasis (partial collapse of lung tissue), which can
then
lead to pneumonia. Frequent position changes with deep breathing and coughing will help prevent development of atelectasis.
Given that this patient is elderly and does not have a diagnosis of secretion retention, vigorous percussion and vibration is not
indicated. Vibration with the head down or standard postural drainage positions will not be tolerated in this elderly patient with
chronic congestive heart failure.

61. Which of the following joint mobilizations would be MOST effective for improving a patient’s ability to progress into
terminal stance?

1. Posterior glide of the talus on the tibia

2. Posterior glide of the calcaneus on the talus

3. Plantar glide of the 1st proximal phalanx on the metatarsal

4. Dorsal glide of the 1st proximal phalanx on the metatarsal

Correct answer: 4
Extension of the 1st metatarsophalangeal joint is needed in terminal stance. The distal metatarsal is convex. The proximal
portion
of the articulating phalanx is concave. According to the convex-concave rule, when a therapist moves a concave joint surface on
a
convex joint surface, the concave joint surface is moved in the same direction as the range-of-motion limitation. Therefore, the
appropriate accessory glide is a dorsal glide of the proximal phalanx on the metatarsal. Posterior glide of the talus on the tibia
and
plantar glide of the 1st proximal phalanx on the metatarsal promote plantar flexion. Posterior glide of the calcaneus on the talus
is
not as relevant for terminal stance, which involves the 1st MTP joint.

62. A physical therapist is examining muscle strength in a patient. The patient is asked to move the leg along the path
indicated by the arrow shown in the photograph and to hold the leg in place while the therapist applies resistance to the leg.
The patient moves through the range of motion shown and is able to take maximal resistance. Intervention should address
which of the following problems?

1. Tightness of the hip medial (internal) rotators

2. Weakness of the gluteus minimus and medius

3. Correction of the right lateral (external) trunk shift

4. Piriformis weakness

Correct answer: 1
The fact that the patient has normal muscle strength of the hip lateral (external) rotators along with limitation of range of
motion
indicates either tightness of the medial (internal) rotators or hip capsule tightness. The gluteus medius and minimus are medial
(internal) rotators of the hip. The trunk shift is a normal substitution pattern found in individuals who lack hip range of motion.
The problem is related to the hip, not the trunk. The patient’s piriformis shows normal strength, so it is not weak.

63. A patient with a complete thoracic spinal cord injury is sitting in a wheelchair on a custom-made cushion. Pressure relief
activities should be performed:

1. when the patient shows signs of pressure sores.

2. every 15 to 20 minutes.

3. every 1 to 2 hours.

4. if the patient does not have an appropriate cushion.

Correct answer: 2
A patient with a thoracic spinal cord level injury is able to perform independent pressure relief strategies and should complete
pressure relief every 15 to 20 minutes.

64. Which of the following lower extremity findings is MOST likely to be the FIRST sign of vascular occlusive disease?

1. Edema of the legs and ankles

14
2. Brown discoloration at the ankles

3. Increased cyanosis when legs are dependent

4. Lack of hair on the toes

Correct answer: 4
Poor hair growth is characteristic for arterial disease due to inadequate cellular nutrition. The first sign of arterial disease is often
loss of hair on the toes. Edema may be present with advanced cases of arterial disease. Increased cyanosis with legs in the
dependent position is present in advanced disease. Brownish discoloration at the ankles is characteristic of chronic venous
disorders.

65. A physical therapist is teaching a patient who has recently undergone knee surgery to use a cane while descending stairs
without a railing. The patient has partial weight-bearing status on the right. Which of the following instructional methods should
the therapist use to teach the patient this activity?
1. The therapist stands behind the patient, and the patient descends with the cane and left leg first.

2. The therapist stands beside the patient, and the patient descends with the right leg first and then the cane.

3. The therapist stands below the patient, and the patient descends with the cane and right leg first.

4. The therapist stands behind the patient, and the patient descends with the left leg first.

Correct answer: 3
Options 1, 2, and 4 are neither safe nor appropriate. When descending stairs, it is generally best for the therapist to be
positioned
below the patient as protection from falling. However, the patient should always descend stairs leading with the involved leg,
the right leg in this case, and the cane.

66. A patient with low back pain also reports constipation, occasional nausea and vomiting, and unexplained weight loss. The
pain worsens when the patient is supine and decreases when the patient leans forward when sitting. The sclerae are yellow.
The signs and symptoms are MOST consistent with which of the following disorders?

1. Appendicitis

2. Cholecystitis

3. Pancreatic carcinoma

4. Irritable bowel syndrome

Correct answer: 3
Pancreatic cancer is characterized by nonspecific and vague symptoms, which can include nausea, weight loss, pain radiating to
the
back or back pain alone, and jaundice. Sitting up and leaning forward may provide some relief by taking pressure off the
pancreas.
Irritable bowel syndrome is characterized by abdominal pain with constipation and diarrhea, nausea and vomiting, but not
referred
back pain or jaundice. Cholecystitis is an inflammation of the gallbladder and typically produces right upper quadrant abdominal
pain.
There may be nausea and vomiting, weight loss, and jaundice. However, the pain is referred to the upper back or right shoulder,
not the low back. There is also anorexia, nausea and vomiting with appendicitis, but the pain is present generally over the right
lower quadrant. Bending forward may worsen the symptoms because of the resultant increase in intraabdominal pressure.

67. Excessive upward rotation of the right scapula is noted when a patient attempts to perform shoulder flexion. Which of the
following exercises is MOST appropriate to help correct the excessive scapular rotation?
1. Right scapular protraction against resistance with the right arm at 90° of flexion

2. Bilateral scapular elevation with the upper extremities at 180° of flexion

3. Wall push-ups with an isometric hold at end range with the elbows extended

4. Bilateral scapular adduction with the upper extremities medially (internally) rotated and adducted across the back

Correct answer: 4
Excessive upward rotation of the scapula can result from weakness of the rhomboids and latissimus dorsi (downward rotators).
The scapular adduction with medial (internal) rotation and adduction of the arm would require action by those muscles.
Option 1 would help strengthen the serratus anterior, an upward rotator of the scapula. Option 2 would activate the upper
trapezius as well as the rhomboids, and, since the upper trapezius is also an upward rotator of the scapula, this would not be
the best exercise to use. Option 3 would also help strengthen the serratus anterior, which would tend to aggravate the problem.

68. Which of the following diagnoses is MOST associated with urinary incontinence?

1. Orchitis

2. Testicular cancer

3. Testicular torsion

4. Benign prostatic hyperplasia

15
Correct answer: 4
The nodular hyperplasia that occurs with benign prostatic hyperplasia causes obstruction of the urethra, resulting in urinary
frequency and urge incontinence. Testicular torsion, orchitis, and testicular cancer are not associated with urinary dysfunction.

69. A physical therapist is working on transfers with a patient who had a brainstem cerebrovascular accident. The patient has
ataxia in all four extremities and a high level of extensor tone in the lower extremities. The patient has fair to good trunk
control. Which of the following transfers is BEST for this patient?

1. Squat pivot

2. Sliding board

3. Standing pivot

4. Dependent tuck

Correct answer: 1
A squat-pivot transfer avoids full lower extremity extension, thus minimizing lower extremity extensor tone. A sliding-board
transfer 

is not appropriate with a high level of ataxia in the upper extremities. A standing-pivot transfer is not appropriate because it
may
increase the extensor tone in the lower extremities. A dependent tuck transfer does not encourage active patient participation or
promote independence.

70. A physical therapist wants to examine the relationship between lower extremity manual muscle test grades and five ranked
categories of functional ambulation ability in a group of older adults. Which of the following statistics is MOST appropriate for
testing this relationship?

1. Paired t test

2. Pearson product-moment correlation (r)

3. Mann-Whitney U test

4. Spearman rho (rs)

Correct answer: 4
The research question is one of relationship or correlation between measures, not comparison of group means, so the t test and
the
Mann-Whitney U test are not appropriate. Because the data for both variables are ranked (ordinal), Spearman rho (Spearman
rank
correlation coefficient) should be used. This is the nonparametric analog of the Pearson correlation coefficient (r). The Pearson r
requires continuous, not ordinal, data.

71. In a research study, a correlation coefficient of .30 was found for the relationship between two variables. Which of the
following interpretations of this finding is MOST appropriate?

1. The variables have a low correlation.

2. Thirty percent of the variability in one variable can be accounted for by the other variable.

3. There are no significant differences between the variables.

4. There is low positive predictive value between the variables.

Correct answer: 1
A correlation coefficient of .26 to .49 is considered low.

72. Which of the following conditions is the MOST likely cause of a reduced vital capacity in a patient who has quadriplegia at
the C5–C6 level?

1. Decreased anterolateral chest expansion resulting from paralysis of the external intercostal muscles

2. Inability of the patient to generate a negative intrapleural pressure secondary to a denervated diaphragm

3. A relatively high resting position of the diaphragm resulting from paralysis of the abdominal muscles

4. Reduced rib-cage elevation due to paralysis of the anterior scalene and sternocleidomastoid muscles

Correct answer: 1
The rib cage would not be able to expand normally during inspiration due to weakness of the external intercostal muscles,
which are innervated by thoracic nerve segments. With a spinal cord lesion at the C5–C6 level, the diaphragm would still receive
innervation from the phrenic nerve (C4). The anterior scalene (C4–C6) would be partially innervated and the sternocleidomastoid
(C2–C3) would be fully innervated. The abdominal muscles would not be innervated since they receive their innervation from
thoracic nerve segments. Paralysis of the abdominal muscles would cause the diaphragm to assume a low resting position.

73. A physical therapist places electrodes on a patient to monitor surface electromyographic activity. The electrode placement
shown in the photograph is MOST appropriate to monitor which of the following muscles?

1. Tensor fasciae latae

16
2. Sartorius

3. Rectus femoris

4. Gluteus minimus

Correct answer: 1
The electrode placement in the photograph is the best placement for the tensor fasciae latae. The electrodes are placed over the
muscle belly and lined up parallel to the muscle fibers. The sartorius is found more distally and medially and following the
direction
of the muscle fibers. The rectus femoris is found more distally and central to the thigh. The gluteus minimus is a deep muscle
that
cannot be easily monitored directly by surface electromyography.

74. A physical therapist places a patient on a strength training program for the lower extremities. The mode of exercise is a
double-leg press unit using free weights. After 1 week, the patient shows a 10-lb (4.5-kg) increase in the amount of weight
the patient is able to lift. What is the MOST likely cause of the patient’s increase in strength?

1. Muscle fiber hypertrophy

2. Neurological adaptation

3. Hyperplasia of the muscle fibers

4. Increase in the amount of actin and myosin

Correct answer: 2
Strength increase in muscle is due to a number of factors, including neurological adaptation and muscle fiber hypertrophy with
an
increase in actin and myosin. Hyperplasia in humans is still controversial. Long-term changes in muscle strength are due to all
of
the factors listed. However, short-term changes, such as changes in 1 week, are most likely to due to neurological factors such
as
more efficient motor unit recruitment, autogenic inhibition, and more efficient co-activation of muscle groups.

75.A physical therapist is considering the use of phonophoresis as part of an intervention plan. Which of the following steps is
the correct FIRST step in the decision-making process to use phonophoresis?

1. Outline the therapeutic goals and outcomes.

2. Select the appropriate coupling agent and medication.

3. Decide on the dosimetry by choosing the appropriate mode and frequency.

4. Determine if there are any contraindications.

Correct answer: 4
Although all of options are involved in the decision-making process, the determination of contraindications is the first thing to
consider, because all other options are unnecessary if the patient has other pathologies that could be a contraindication.

76. A physical therapist is evaluating a patient who has shoulder pain. The patient notices the shoulder pain at work when
stocking shelves that are overhead. The pain is not apparent when stocking shelves at waist or chest level. The patient MOST
likely has weakness in which of the following muscles?

1. Pectoralis minor

2. Upper trapezius

3. Deltoid

4. Rhomboid major

Correct answer: 2
Weakness in the pectoralis minor would not cause restriction of the scapula but would likely cause scapular hypermobility.
Weakness in the upper trapezius would decrease upward rotation of the scapula during shoulder flexion and abduction. The
more
the shoulder is elevated, the more noticeable this would be. The decreased scapular movement would increase the
predisposition
toward impingement. Weakness in the deltoid would cause the humerus to move downward, not upward, during shoulder
elevation.
Weakness in the rhomboid major would not cause restriction of the scapula, but would likely cause scapular hypermobility.

77.A patient who sustained a mild cerebrovascular accident 3 weeks ago is being prepared by a physical therapist for
discharge to home and an adult day program. To facilitate the discharge plan, the MOST appropriate health professional for
the therapist to consult with is the:

1. skilled nursing coordinator.

2. occupational therapist.

17
3. medical social worker.

4. primary physician.

Correct answer: 3
Medical social service staff deal with home situations and financial supports and act as a resource director on behalf of the
patient. Social services would be the most appropriate choice to help arrange and coordinate rehabilitative services for the
patient while the patient is at home. Although nurses, occupational therapists, physical therapists, and physicians may all be
involved with direct patient care, they would not be the most appropriate for this level of discharge planning.

78. In a research study, a physical therapist performs the same goniometric measurement on the same group of control
subjects during two consecutive testing sessions. This process is used to demonstrate which of the following measurement
properties?

1. External validity

2. Instrument reliability

3. Intrarater reliability

4. Interrater reliability

Correct answer: 3
Intrarater reliability describes the repeatability of measurements made by one person (repeatability within that person).

79. Sensory-level electrical stimulation is MOST appropriate for a patient with which of the following conditions?

1. Chronic low back pain of somatic origin

2. Acute ankle sprain with edema

3. Supraspinatus tendonitis

4. Active Raynaud syndrome

Correct answer: 4
Regarding chronic low back pain of somatic origin, the patient has a chronic problem, so will most likely require motor-level
stimulation, because it provides a longer-lasting analgesia. There is no evidence to indicate that sensory-level stimulation is
effective in the treatment of edema. For edema reduction, rhythmic muscle contraction is preferred. Iontophoresis is more
appropriate than sensory-level electrical stimulation for treatment of a tendonitis, because it involves the delivery of anti-
inflammatory medications. Raynaud syndrome is a condition in which the smallest arteries that bring blood to the fingers or toes
constrict when exposed to cold or from an emotional upset. Sensory-level stimulation over nerve roots and trunks can increase
peripheral vasodilatation.

80. A patient with which of the following diagnoses would MOST likely benefit from pursed-lip breathing during exercise?

1. Peripheral vascular disease

2. Congestive heart failure

3. Emphysema

4. Sarcoidosis

Correct answer: 3
Peripheral vascular disease is a vascular problem, not a pulmonary problem. Congestive heart failure may lead to pulmonary
problems, but not obstructive problems. Emphysema is an obstructive lung disease, for which pursed-lip breathing may be
beneficial. Sarcoidosis is a restrictive lung disease for which pursed-lip breathing is not beneficial.

81. A patient has a history of neck pain that is aggravated by long periods of sitting and becomes progressively worse by
evening. Range of motion and strength of the neck and shoulders are within normal limits. Sensation and reflexes are intact in
both upper extremities. The patient has a forward head and excessive thoracic kyphosis. The MOST appropriate exercise
program should focus on:

1. stretching of the neck flexors and pectoral strengthening.

2. upper trapezius strengthening and pectoral stretching.

3. pectoral strengthening and rhomboid stretching.

4. rhomboid strengthening and axial neck retraction.

Correct answer: 4
The history suggests that prolonged improper positioning of the cervical spine resulted in neck pain. A chronic forward head and
kyphosis results in hyperextension in the upper cervical spine and excessive flexion in the upper thoracic spine. Further muscle
length adaptation occurs with tight anterior muscles and stretched posterior muscles. Treatment should include correction of
muscle
weakness or imbalance. Rhomboid strengthening and axial neck extension are the only options that are both correct.

82. A patient suspected of having hypoglycemia is MOST likely to show which of the following signs?

18
1. Fruity smelling breath

2. Thirst, nausea, and vomiting

3. Dry, crusty mucous membranes

4. Difficulty speaking and concentrating

Correct answer: 4
A common mental state manifestation of hypoglycemia is difficulty speaking and concentrating, whereas in hyperglycemia there
is outright confusion. The other options are all signs of hyperglycemia and not signs of hypoglycemia.

83. A patient is limited in shoulder abduction, as shown in the displayed radiograph. Which of the following mobilization
techniques is MOST likely to assist the patient in achieving increased shoulder abduction?

1. Posterior glide

2. Anterior glide

3. Inferior glide

4. Superior glide

84. Which of the following assignments is MOST appropriate for a physical therapist to delegate to a volunteer?

1. Restocking treatment booths with linens, ultrasound gel, and massage lotion

2. Attending a patient who is on a tilt table while the therapist takes a phone call

3. Transporting a patient who reports dizziness back to the patient's room

4. Transferring a patient from the mat table to a wheelchair

Correct answer: 1
The restocking of treatment booths with supplies is the only option that does not involve direct patient contact or care and thus
is
the MOST appropriate activity to delegate to a volunteer. Although volunteers may at times be involved with patient care
activities
(i.e., transporting patients), the patients in the situations described in options 2, 3, and 4 are at potential risk and would require
supervision by someone other than a volunteer.

85. A physical therapist is teaching a motor skill to a patient with chronic hemiplegia. Which of the following teaching
approaches should be MOST emphasized?

1. Habituation

2. Sensitization

3. Compensatory strategy

4. Recovery of normal movement

Correct answer: 3
A patient with chronic hemiplegia is unlikely to recover normal function. Compensatory strategies are used when there is a
permanent loss of function which prevents reacquiring normal movement patterns. Habituation is a decrease in responsiveness
that occurs as a result of repeated exposure to a nonpainful stimulus. In the acute patient, the emphasis is on recovery of
normal
function, but this patient has a chronic condition. Sensitization is an increased responsiveness following a threatening or
noxious stimulus.

86. A patient has low back and leg pain, with symptoms extending to the bottom of the foot. During the physical therapy
examination, the patient does not report leg pain in the first test position (photograph #1) but reports a severe increase in
symptoms in the second test position (photograph #2). Which of the following conclusions is MOST likely?

1. The pain is the result of a herniated disc.

2. The patient may be displaying nonorganic symptoms.

3. The symptoms are the result of an inflamed sciatic nerve.

4. The hamstrings are in a facilitated state of contraction.

Correct answer: 2
Because sitting knee extension and the straight-leg raise culminate in essentially identical positions, symptomatic responses to
the
two types of maneuvers should be similar. If the patient had a symptomatic herniated disc, both positions would result in a
similar
symptom increase. If the patient had an irritated sciatic nerve, both positions would result in a similar symptom increase.
If the patient had a facilitated hamstring, both positions would result in a similar symptom response.

87. A patient is referred for recommendations regarding purchase of a wheelchair. The measurements of the patient while
sitting are 16 in (40.6 cm) across the widest point of the hips and 18 in (45.7 cm) from the rear of the buttocks to the
19
87. A patient is referred for recommendations regarding purchase of a wheelchair. The measurements of the patient while
sitting are 16 in (40.6 cm) across the widest point of the hips and 18 in (45.7 cm) from the rear of the buttocks to the
popliteal crease. Which of the following wheelchair dimensions would be BEST suited for this patient’s needs?

1. Seat width and seat depth of 18 in (45.7 cm)

2. Seat width of 18 in (45.7 cm) and seat depth of 16 in (40.6 cm)

3. Seat width and seat depth of 16 in (40.6 cm)

4. Seat width of 16 in (40.6 cm) and seat depth of 18 in (45.7 cm)

Correct answer: 2
The seat width should be slightly wider than the width of the widest body part, and the depth should come to within 1 inch of the
popliteal fossa.

88. A physical therapist is initiating intervention with a postoperative patient who is taking 20 mg of oral oxycodone
(Oxycontin) for pain relief. In planning an intervention for this patient, the physical therapist should anticipate that:

1. the patient may be hypertensive.

2. the medication may trigger cardiac arrhythmias in the patient.

3. the patient will have an increased likelihood for developing diarrhea.

4. the patient may demonstrate respiratory depression.

Correct answer: 4
Regarding the patient being hypertensive, the opposite is likely. Orthostatic hypotension is a potential side effect. Arrhythmias
are not listed as a recognized side effect of opioids. Regarding developing diarrhea, gastrointestinal motility is decreased, so the
opposite effect of constipation is a frequent problem. Opioids tend to make the medullary chemoreceptors less responsive to
carbon dioxide, thus slowing down respiratory rate and inducing a relative hypoxia and hypercapnia. The respiratory response to
exercise may be blunted.

89. During which of the following scenarios would gloves be required to comply with standard precautions?

1. During all patient care in the hospital setting

2. Performing range of motion on a patient with acquired immunodeficiency syndrome

3. Massaging the neck of a patient with hepatitis C

4. Changing an infant’s diaper in a pediatric setting

Correct answer: 4
Gloves are required only during contact with blood or body fluids, not during patient care that does not involve coming in contact
with blood or body fluids. Gloves are required whenever changing an infant’s diapers, because this activity involves coming into
contact with body fluids.

90. Radiograph question.

91. Which of the following descriptions accurately reports a NORMAL patient response to deep tendon reflex testing?

1. 2 or plus (+)

2. 3 or plus (++)

3. 0 or minus (-)

4. 1 or minus (-)

A normal response is 2 or plus (+). Minus (-) or 0 indicates an absent reflex, and minus (-) or 1 indicates a diminished response.
Plus (++) or 3 indicates an exaggerated response, and plus (+++) or 4 indicates clonus.

92. A patient presents with weakness throughout the right lower extremity with normal strength on the left side. Sensory
testing shows a loss of pressure sensation over the right thigh and leg and a loss of pain and temperature sensation over the
left thigh and leg. The patient has a positive Babinski sign on the right. Which of the following associated findings is MOST
likely to be found during further examination of this patient?

1. The presence of clonus in the left ankle

2. Marked atrophy in the right lower extremity muscles

3. Spasticity in the left lower extremity

4. Increased deep tendon reflexes on the right side

Correct answer: 4

20
The presence of clonus would be on the right side, not the left side. Spasticity would be on the right side, not the left side. This
is a
case of hemisection of the spinal cord, Brown-Séquard syndrome. The lesion is on the right side of the spinal cord, and it is an
upper
motor neuron lesion (positive Babinski sign) with damage to the corticospinal tract, posterior columns, and lateral spinothalamic
tract.
Upper motor neuron spinal cord lesions present with hyperactive tendon reflexes (in this case on the right side), clonus, and
spasticity ipsilateral to the side of the lesion. Pressure sensation is lost on the ipsilateral side, and pain and temperature are lost
contralateral
to the side of the lesion due to the crossing of those fibers in the spinal cord.

93. While a physical therapist is performing transfer training from bed to chair with a patient who had a total knee arthroplasty
2 days ago, the electrocardiograph monitor alarms and the therapist notes that premature ventricular contractions have
developed. What is the therapist’s BEST course of action at this time?

1. Position the patient on a stable surface and discontinue physical therapy for the day.

2. Continue the transfer to the chair and monitor the patient’s oxygen saturation levels.

3. Continue the transfer to the chair and immediately notify the nurse about the premature ventricular contractions.

4. Position the patient on a stable surface and determine the stability of the premature ventricular contractions.

Correct answer: 4
Option 1 would apply only after the therapist has done further analysis of the situation. Monitoring oxygen saturation is useful
but
not the most immediate thing to monitor. The therapist should monitor the electrocardiogram and blood pressure of the patient.
Although notifying the nurse is something the therapist should do, this should be done after an initial evaluation of the stability
of
premature ventricular contractions. Onset of premature ventricular contractions can be benign or stable. Less than six
contractions/minute is generally stable, while greater than six contractions/minute is considered less stable. A physical therapist
should be able to determine this stability.

94. Which of the following postural drainage positions is MOST appropriate for a patient who has aspiration pneumonia in the
right middle lobe and who had a craniotomy 2 days ago?

1. Right sidelying with one-quarter turn toward supine

2. Left sidelying with one-quarter turn toward supine with head of bed down approximately 20°

3. Right sidelying with one-quarter turn toward supine with head of bed down approximately 20°

4. Left sidelying with one-quarter turn toward supine

Correct answer: 4
Option 1 is incorrect because the traditional position to drain the right middle lobe is left sidelying, not right sidelying. Option 2
is the traditional drainage position for right middle lobe, but does not take into account the intracranial pressure issues. Option
3 is the traditional drainage position for right middle lobe but does not take into account the intracranial pressure issues. The
traditional position to drain the right middle lobe is left sidelying with one-quarter turn toward supine, head of bed down
approximately 20°. However, because of the recent craniotomy, the patient’s increased intracranial pressure is a major issue
to monitor and keep stable. Therefore, this traditional position should be modified to flat sidelying, as described in option 4.

95. To help students apply a newly learned skill to clinical practice, the MOST effective action for the clinical instructor to take
is to:

1. point out possible clinical situations and discuss how the skill would apply to them.

2. have the students research reference materials and compile a list of the steps required to acquire the skill.

3. prepare a list of indications and contraindications for the skill.

4. have the students provide examples of clinical situations where the skill would be appropriately applied.

Correct answer: 4
Behavioral objectives should be learner centered, outcome oriented, specific, and measurable. Option 4 is the only one that is
learner (student) centered and specific to a situation. Options 1 and 3 require action by the clinical instructor, not the student.
Option 2,
which may be a step in the process, is not as learner centered or outcome centered as Option 4.

96. During the shoulder examination of a patient, a physical therapist notes the presence of a capsular pattern without
radicular pain. To help establish the cause of the capsular pattern, the therapist should NEXT:

1. perform axial compression on the cervical spine to check for nerve root compression.

2. ask the patient if there has been any prior trauma to the shoulder joint.

3. check for a painful arc during active range of motion.

4. examine the shoulder for a rotator cuff tear.

Correct answer: 2

21
A nerve root impingement commonly gives radicular pain. One of the most common causes of a capsular pattern in the shoulder
is
traumatic arthritis following injury to the shoulder. The capsular pattern may gradually develop over time. A painful arc is often
associated with a noncapsular pattern such as seen with bursitis or tendonitis at the shoulder. A torn rotator cuff is associated
with
weakness and loss of active shoulder elevation.

97. A patient had knee surgery 4 weeks ago. Which of the following neuromuscular electrical stimulation parameters would be
MOST appropriate to use for strengthening the patient’s quadriceps muscle?

1. 1 to 4 pps, 100 microseconds

2. 1 to 4 pps, 350 microseconds

3. 40 to 50 pps, 350 microseconds

4. 100 pps, 100 microseconds

Correct Answer: 3
The pulse width of 100 microseconds makes it difficult to achieve a strong enough motor response. A 1 to 4 pps frequency may
lead to a twitch motor response, but not tetany, which is needed for strengthening. The correct answer requires a frequency that
will lead to tetany (something above about 30 Hz) and a long enough pulse width to recruit motor fibers (usually something
greater than 200 microseconds). Therefore, of the options, 40 to 50 pps with a pulse width of 350 microseconds is the best.
Option 4 describes the typical parameter for conventional transcutaneous electrical nerve stimulation for acute pain
management. However, the pulse width of 100 microseconds makes it difficult to achieve a motor response, and the higher
frequency of 100 pps leads to rapid fatigue.

98. A patient is asked to grip a white card between the thumb (1st digit) and index finger (2nd digit) with both hands. The
physical therapist pulls on the card in the direction of the arrow shown. As the therapist pulls on the card the patient’s right
thumb flexes at the interphalangeal joint. The results indicate weakness in which of the following muscles?

1. Flexor pollicis longus

2. Abductor pollicis brevis

3. Adductor pollicis

4. Extensor pollicis longus

Correct answer: 3
The test shown in the photograph is the Froment test. Both thumbs (1st digits) should stay extended during the test. If the
thumb
(1st digit) flexes, it is indicative of weakness of the adductor pollicis with substitution by the flexor pollicis longus, which is
usually
due to a lesion of the ulnar nerve.

99. During the gait evaluation of a patient, a physical therapist notices that the patient laterally bends excessively toward the
right side during the midstance phase on the right. For the therapist to test the suspected muscle for Normal strength (5/5),
the patient should FIRST be positioned:

1. prone with the knee straight.

2. supine with the knee bent.

3. seated with the hip flexed to 110°.

4. sidelying on the left side.

Correct answer: 4
Prone with the knee straight would be used to test for a Poor (2/5) grade. Supine with knee bent is used to isolate the gluteus
maximus. Seated hip flexion is used to test the hip flexor muscle strength. The most likely cause of laterally bending toward the
stance limb is abductor weakness on the stance side. The lateral bending helps compensate for weak abductors. The right
abductors
would be tested from a left sidelying position for Fair (3/5) or greater strength.

100. A patient with a cerebellar stroke has received functional balance training for 4 weeks. Which of the following tests is
MOST appropriate to measure the effectiveness of the physical therapy intervention?

1. Romberg Test

2. Berg Balance Scale

3. Fugl-Meyer Assessment

4. Barthel Index

Correct answer: 2
Although the Romberg Test is a measure of the role of vision in balance, it alone would not be the most appropriate measure for
functional balance. The Berg Balance Scale is an objective measure of static and dynamic balance abilities and consists of 14
commonly performed functional tasks; therefore, it is the most appropriate tool to use as a measure of intervention
effectiveness.

22
The Fugl-Meyer Assessment is appropriate for cortical strokes and would not be the most appropriate for a cerebellar stroke. It
is
not as comprehensive in balance tasks as the Berg Balance Scale. The Barthel Index is a more global instrument and is not as
focused on functional balance as the Berg Balance Scale.

101. A patient in the eighth month of pregnancy has numbness and tingling of the left hand, except for the little finger (5th
digit). She demonstrates edema of the hand and fingers, a positive Tinel sign at the wrist, and a Good (4/5) muscle test grade
of the wrist and finger flexors. The MOST appropriate intervention is:

1. a wrist splint to position the wrist in full extension.

2. a hot pack followed by tendon gliding exercises.

3. resistive exercises for the wrist and finger flexors.

4. frequent rest and elevation of the left upper extremity.

Correct answer: 4
Compression on the median nerve (carpal tunnel syndrome) is occurring, most likely as a result of swelling associated with the
individual being in the eight month of pregnancy. In this case, rest and elevation would do the most to decrease the edema and
relieve the symptoms. The wrist should not be positioned in full extension. Initial conservative treatment sometimes includes
cock-up splinting to hold the wrist in neutral to 10° of extension, but not full extension. Although tendon gliding exercises may
be used,
heat would not be indicated since it may increase the edema. Resistive exercises for the wrist and fingers may aggravate the
compression in the carpal tunnel.

102. Which of the following locations corresponds to the sensory function of the nerve root exiting between the third and
fourth lumbar vertebrae?

1. Lateral knee joint line

2. Medial knee joint line

3. Plantar aspect of the heel

4. Dorsal web space between the first and second toes

Correct answer: 2
The L3 nerve root exits between the third and fourth lumbar vertebrae and supplies sensory information from the region of the
medial knee joint line.

103. An inpatient physical therapy department has only one physical therapist and one physical therapist assistant on duty,
due to staff illness. Treatment of a patient with which of the following conditions and circumstances is MOST appropriate for
the therapist to delegate to the assistant?

1. Ataxia, who is undergoing a trial to determine an appropriate assistive device

2. Hemiparesis, whose initial evaluation has not been completed

3. Multiple sclerosis, who is receiving gait training with a rolling walker

4. Alzheimer disease, who is easily agitated and is receiving initial gait training

Correct answer: 3
A physical therapist would be required to perform the initial evaluation, plan of care, reevaluations, modifications to
the plan of care, and discharge plans for the patient. The patient who is stable and has an existing program would be the most
appropriate patient to be delegated to the assistant. Therefore, the patient who has multiple sclerosis and who has an
established
gait training program would be the most appropriate. The other three patients are receiving their initial treatments.

104. When considering special tests for orthopedic assessment, selection of a test with adequate validity is important because
the:

1. results of the test can be standardized.

2. test measures what it is supposed to measure.

3. results of the test are reproducible.

4. test can be accurately performed by someone else.

Correct answer: 2
Option 2 is the definition of validity. Options 3 and 4 refer to reliability measures. Option 1 is incorrect because results can be
standardized even though they are not valid.

105. A patient who recently had bilateral transtibial amputations wants to have a ramp built to travel from the back deck of his
house to the pool in his wheelchair. The vertical distance from the door to the ground level is 5 ft (1.5 m). Which of the
following ramp specifications is BEST for the patient?

1. One continuous ramp, 60 ft (18 m) long

23
2. One continuous ramp, 30 ft (9 m) long

3. Two ramps, each 60 ft (18 m) long, connected by a level area

4. Two ramps, each 30 ft (9 m) long, connected by a level area

Correct answer: 4
One continuous ramp that is 60 ft (18 m) long is too long of a run without a level area to rest. One continuous ramp that is 30 ft
(9 m) long is too steep a slope. Two ramps, each 60 ft (18 m) long, even if connected by a level area, would be unnecessarily
long.
The rise should not be greater than 30 in (76 cm) for any one run and should be 1 ft (.3 m) in length per each 1 in (2.5 cm)
rise.

106. A physical therapist is developing an exercise program for an older adult to improve routine health maintenance and
fitness. The patient has a history of a right ankle fracture 15 years ago, aortic aneurysm repair 3 years ago, and open-heart
surgery for a valve replacement 6 years ago. Which of the following activities is CONTRAINDICATED?

1. Aquatic exercise for 30 minutes

2. Cycling at 60% of maximum heart rate

3. Endurance exercise using basic calisthenics

4. Weight lifting at 50% of maximum voluntary contraction

Correct answer: 4
Resistive exercise is contraindicated for a patient with a history of aortic aneurysm, even after surgical repair.

107. Which of the following home programs is MOST appropriate for a patient with chronic lateral epicondylitis?

1. Using a forearm cuff to increase loading on the extensor tendons

2. Performing exercises for wrist strength and stretching

3. Administering iontophoresis with dexamethasone (Decadron) and lidocaine (Xylocaine)

4. Doing friction massage of the brachioradialis tendon

Correct answer: 2
Lateral epicondylitis is caused by overuse of the wrist extensors that originate on the lateral epicondyle of the humerus,
especially the extensor carpi radialis brevis. If the lateral epicondylitis is at a chronic stage, conditioning of the extensor muscles
and sustained grip activities will be most effective in long-term management. A forearm cuff is thought to decrease the muscle
loading. Iontophoresis
would not be appropriate for a home program. Friction massage of the brachioradialis would not be appropriate since the
extensor
carpi radialis brevis muscle is usually the one affected.

108. A patient is receiving mechanical intermittent cervical traction with an on/off duty cycle of 20 seconds/10 seconds. The
patient reports increased pain each time the traction unit cycles on, which then subsides over the duration of the on time.
Which of the following modifications to the duty cycle is MOST appropriate?

1. Increase the off time to 20 seconds.

2. Increase the on time to 25 seconds.

3. Decrease the off time to 5 seconds.

4. Decrease the on time to 15 seconds.

Correct answer: 1
When patients have severe symptoms, it is often useful to have increased on and off times to reduce the movement caused by
the
cycling. Increasing the off time allows for longer recovery time prior to re-initiating the traction and reduces the cycling. As the
pain symptoms decrease, the relaxation time can be decreased. Options 2, 3, and 4 do not allow for a longer off or recovery
period.

109. Which of the following arterial blood gas readings indicates acidemia?

1. 7.3

2. 7.4

3. 7.45

4. 7.5

Correct answer: 1
Normal range of arterial blood gas is a pH between 7.35 and 7.45. A pH of 7.35 or lower indicates acidemia, also called an
acidotic state.

110. A patient with testicular cancer is most likely to report which of the following signs or symptoms INITIALLY?
24
110. A patient with testicular cancer is most likely to report which of the following signs or symptoms INITIALLY?

1. Back pain

2. Pain referred to the groin

3. Swelling of superficial lymph nodes

4. Diffuse testicular pain and swelling

Correct answer: 4
The most common initial sign is enlargement of the testis with diffuse testicular pain, swelling or hardness. Back pain may be
present, as well as pain referred to the groin or swelling of superficial lymph nodes, but testicular pain and swelling are the most
common predominant symptoms initially.

111. Which of the following positions is BEST to assess the length of a patient’s rectus femoris muscle?

1. Sidelying with tested hip in flexion

2. Supine with tested hip and knee in flexion

3. Prone with tested knee in flexion

4. Sidelying with tested hip in extension

Correct answer: 3
Option 1 does not mention the knee position and has the hip flexed, which shortens the rectus femoris. Option 2 has the hip
flexed,
which shortens the rectus femoris. Prone with the knee in flexion keeps the hip in neutral and does not allow the hip to flex. This
position is used in the Ely test. Option 4 has the hip in the correct position, but does not mention the knee position.

112. What is the closed packed position of the wrist?

1. Flexion with ulnar deviation

2. Flexion with radial deviation

3. Extension with ulnar deviation

4. Extension with radial deviation

Correct answer: 4
The closed packed position for the radiocarpal joint is full extension with radial deviation. The closed packed position for the
midcarpal joint is extension with radial deviation.

113. A patient fell while rock climbing 2 days ago. The fall resulted in a fracture of the right ankle, requiring an open reduction
and internal fixation. The patient also sustained a nerve injury that resulted in significant weakness in the muscles in the right
C6–C7 myotome. The patient is restricted to non-weight-bearing status on the right lower extremity. Which of the following
assistive devices is MOST appropriate for the patient?

1. Axillary crutches

2. Wheelchair

3. Standard walker

4. Forearm crutches

Correct answer: 2
The axillary crutches, standard walker, and forearm crutches all require adequate strength in the elbow extensors and latissimus
dorsi, especially due to the non-weight-bearing status of the patient. The non-weight-bearing status on the right could indicate
any of the options; however, because the patient has weakness of the muscles in the C6–C7 dermatome, the patient would have
difficulty using any assistive device that requires use of the elbow extensors and latissimus dorsi muscles. Therefore, the best
option
is the wheelchair, because it provides mobility.

114. A physical therapist is examining the feet of a patient with type 2 diabetes. Which of the following tests is BEST to
determine the patient’s risk for developing foot ulceration?

1. Pain sensation

2. Pressure threshold

3. Two-point discrimination

4. Temperature awareness

Correct answer: 2
Pressure thresholds using nylon filaments are the most sensitive and specific. Several studies support the use of the 10-gram
(Semmes-Weinstein 5.07) nylon filament as the threshold for protective sensation. Patients unable to feel a 10-gram nylon
filament

25
are considered unable to protect their feet from injury and are at risk of ulceration.

115. A physical therapist is planning a three-session educational series on physical therapy evaluation of the low back for
fourth-year medical students. Which of following activities is MOST important to include in the first session?

1. An overview of the material that will be covered in all three sessions

2. A pretest to determine the level of knowledge of the participants

3. Active participation of the students in a low back evaluation

4. A complete demonstration of a low back evaluation

Correct answer: 2
An overview presented on the first day would imply that the therapist is not going to modify the series to the students’ current
knowledge level, which may be ascertained from the pretest. It is most important for the physical therapist to know what level
of
knowledge the students have. Although active participation is important for learning in every session, neither it, nor a complete
demonstration of a low back evaluation, is most important to include in the first session.

116. A physical therapist reads in a report that a child has a standard score of -2.0 on a measure of development. Which of
the following interpretations of this score is BEST?

1. The child’s score is well below the mean score, compared to other children’s scores.

2. The child’s score is close to the mean score, compared to other children’s scores.

3. The child is doing better on this measure than approximately 85% of children.

4. The child is doing worse on this measure than approximately 85% of children.

Correct answer: 1
A standard score, or z-score, relates to the number of standard deviation units a score is above or below a mean. Standard
scores
are often used in developmental measures because they are a useful comparison to a larger or normal group. A standard score
of
-2.0 is well below the mean because two standard deviation units below the mean indicates that only 2.27% of children have
scored
lower or, stated conversely, that 97.73% of children scored better. At -1.0 standard deviation units, 84.14% of children scored
better.


117. A patient with an L4–L5 posterolateral herniated nucleus pulposus is MOST likely to have sensory deficits in which of the
following locations?

1. Medial knee

2. Over the popliteal fossa

3. Plantar aspect of the fifth toe

4. Dorsum of the great toe

Correct answer: 4
The medial knee is the L3 dermatome. The popliteal fossa is S2 dermatome. The plantar foot is S1 or S2. A posterolateral disc
bulge at
L4-L5 will most likely affect the L5 nerve root and the dermatome for the L5 nerve root. The L5 dermatome includes the dorsal
aspect of the great toe. The dorsal area of the great toe is consistently included in the L5 dermatome.

118. A 72-year-old patient reports bilateral calf pain which gets progressively worse while walking. The pain episodes have
been gradually increasing in frequency and severity, after an insidious onset 2 years ago. Sitting decreases the patient’s
symptoms. Which of the following pathologies is the MOST likely cause of the patient’s leg pain?

1. Lumbar foraminal stenosis

2. Gluteal artery claudication

3. Herniated nucleus pulposus

4. Deep vein thrombosis

Correct answer: 1
Lumbar stenosis by definition causes lower extremity symptoms with extension activities, and stenosis is more common in an
older population. Gluteal artery claudication would cause buttock pain, not calf pain. This patient is relatively too old for a
herniated
nucleus pulposus, and most cases of herniated nucleus pulposus do not involve bilateral symptoms. Nothing given in the
patient’s
history would increase the likelihood of a deep vein thrombosis.

119. A patient with Guillain-Barré syndrome who is experiencing progressive paralysis is admitted to an intensive care unit.
Which of the following interventions is MOST appropriate for a physical therapist to perform as a member of the
26
119. A patient with Guillain-Barré syndrome who is experiencing progressive paralysis is admitted to an intensive care unit.
Which of the following interventions is MOST appropriate for a physical therapist to perform as a member of the
interdisciplinary team managing the patient’s care?

1. Alter the patient’s mechanical ventilation settings.

2. Develop the patient’s medication schedule.

3. Perform airway clearance techniques.

4. Discuss the patient’s medical prognosis with the family.

Correct answer: 3
The key phrase in this question is “interdisciplinary.” Of all the options provided, only airway clearance is within the scope of
practice for physical therapists. Patients with Guillain-Barré syndrome may experience respiratory muscle fatigue or paralysis
and are susceptible to pulmonary infection. Pulmonary hygiene is a critical role in their care.

120. A physical therapist examining wrist-joint play finds restriction in the direction indicated by the arrow. The therapist
should suspect a decrease in which joint motion?

1. Radial deviation

2. Ulnar deviation

3. Flexion

4. Extension

Correct answer: 1
The therapist is shown performing an ulnar glide, which is the same joint motion used for radial deviation. Limited motion in this
direction indicates limited ability to perform radial deviation.

121. Which of the following nervous system complications would indicate the poorest prognosis for a patient with acquired
immunodeficiency syndrome (AIDS)?

1. Toxoplasmosis

2. Leukoencephalopathy

3. Myelopathy

4. Polyneuropathy

Correct answer: 2
Although all of the options are serious complications in patients with AIDS, multifocal leukoencephalopathy is the most serious,
and death occurs in a few months after it is diagnosed.

122. Which of the following examination findings is consistent with dehydration?

1. Poor turgor

2. Dependent edema

3. Pitting edema

4. Hypertension

Correct answer: Poor skin turgor is one of the signs of dehydration. The skin, when lifted up between the fingers, does not
return to its original position in a rapid manner.

123. A patient who is a waitress reports weakness in the right hand that causes her to drop dishes at work whenever she is
distracted. The patient also reports wrist and hand pain and numbness over the thenar eminence, both of which occur during
the day and at night. The patient’s thenar muscle strength is Poor (2/5) throughout. Which of the following long-term physical
therapy goals is MOST appropriate for this patient?

1. The goal is to increase tactile sensitivity in 4 weeks, so that the patient is able to hold objects and not drop dishes
at work.

2. The patient will demonstrate decreased pain in the wrist in order to be able to sleep at night.

3. The long-term goal is to increase strength in the thenar muscles and decrease pain in the wrist back to normal
levels.

4. The patient will demonstrate thenar muscle strength of Good (4/5) grade in 6 weeks and be able to hold objects in
hand while at work.

Correct answer: 4
A well-written goal includes an audience (who), behavior (what will be done), condition (under what circumstances), and
degree (amount of change). The only option that includes all of these conditions is option 4, “The patient (who) will demonstrate
thenar muscle strength (behavior) of Good (4/5) grade (degree) and be able to hold objects in hand while at work (condition).”
Options 1, 2, and 3, are each missing one or more of the required elements.

27
124. A patient with a severe cerebellar lesion can sit independently, can stand with minimal assistance, and requires moderate
assistance of one person to walk safely. The patient wants to transfer independently from a wheelchair to a bed. Which of the
following transfer techniques is MOST appropriate for the patient?

1. Standing pivot

2. Manual pivot

3. Mechanical lift

4. Sliding

Correct answer: 4
Standing-pivot transfer requires standing, which the patient is not able to perform safely. Manual-pivot transfer involves the
assistance of another person, so would not allow for independence. Mechanical-lift transfer involves the assistance of another
person, so would not allow for independence. Sliding transfer is the safest independent technique, because the patient cannot
stand
without minimal assistance.

125. A physical therapist is examining a patient who has unilateral lower extremity pain when walking. The pain is relieved by
rest. Which of the following descriptions of symptom behavior reported by the patient will help the therapist confirm a
diagnosis of intermittent claudication?

1. Pain relief upon forward bending or sitting

2. Pain relief upon standing

3. Cramping pain that occurs at a predictable distance walked

4. Numbness and tingling that occurs at a predictable distance walked

Correct answer: 3
Pain relief upon forward bending or sitting and pain relief upon standing are more diagnostic for lumbar origin of pain. Cramping
pain
that occurs at a predictable distance walked is a common presentation and clinical manifestation of vascular limitation.
Numbness and
tingling that occurs at a predictable distance walked are more indicative of a neurologic cause of pain rather than a vascular
cause.

126. A patient reports dizziness and blurred vision when walking and turning the head to either the left or the right. The
patient has no problem when the head is kept still while walking. Which of the following systems is MOST likely involved?

1. Visual

2. Vestibular

3. Somatosensory

4. Musculoskeletal

Correct answer: 3
Since the patient does not have symptoms while looking straight ahead, the visual acuity (peripheral visual system) does not
appear
to be at fault. A patient who reports visual blurring with head motion is experiencing oscillopsia. Oscillopsia is a visual instability
with
head movement in which images appear to move or bounce. It is often due to decreased vestibulo-ocular reflex. No
information is
given in the question to suggest that the patient is having difficulty with somatosensory input or that indicates musculoskeletal
deficits.

127. A physical therapist is preparing for gait training with a young adult patient with paraplegia. Which of the following gait
training options is MOST appropriate for the patient’s first session?

1. Swing-through gait pattern with a walker

2. Swing-through gait pattern with forearm crutches

3. Swing-to gait pattern with axillary crutches

4. Swing-to gait pattern in the parallel bars

Correct answer: 4
Option 1 is an inappropriate assistive device for a young adult with paraplegia. Option 2 is the ultimate goal of gait training but
would
be too difficult for the patient’s first attempt. Option 3 is not the most efficient gait aide for a patient with paraplegia. The first-
time
session of gait training for a patient with paraplegia should be in the parallel bars. A swing-to gait pattern would be the easiest
for the
patient to learn initially.

128. A physical therapist is treating a patient with chronic range of motion limitation due to tight hamstrings. Which of the
28
128. A physical therapist is treating a patient with chronic range of motion limitation due to tight hamstrings. Which of the
following applications of ultrasound and stretching is BEST to restore normal range of motion?

1. Pulsed ultrasound at 1 MHz with onset of stretching 10 minutes after the ultrasound treatment

2. Pulsed ultrasound at 1 MHz with stretching for 10 minutes during and immediately after the ultrasound treatment

3. Continuous ultrasound at 1 MHz with onset of stretching 10 minutes after the ultrasound treatment

4. Continuous ultrasound at 1 MHz with stretching for 10 minutes during and immediately after the ultrasound
treatment

Correct answer: 4
Stretching should be done during and immediately after the ultrasound treatment. 1 MHz continuous ultrasound provides the
greatest
thermal effects and would facilitate the stretch.

129. During an initial physical therapy evaluation, a patient reports occasional breathlessness. Which of the following patient
reports indicates the PRIMARY need for a more thorough pulmonary examination by the physical therapist?

1. There is paroxysmal nocturnal dyspnea.

2. The symptoms are relieved by leaning forward.

3. There has been a recent change in physical activities.

4. The breathlessness is associated with light-headedness.

Correct answer: 2
Paroxysmal nocturnal dyspnea is common in severe heart disease. Relieving the breathlessness symptoms by a positional
change is suggestive that the problem is pulmonary in origin. Occasional breathlessness may be expected with a change in
physical activities,
but more history should be taken to determine if the change is expected. Light-headedness is more indicative of a cardiac
problem.

130. A 90-year-old hospitalized patient was referred for physical therapy evaluation and intervention following a C5 fracture
secondary to a fall. The patient describes neck pain and left knee pain but reports no other postinjury changes in the
extremities. The rehabilitation prognosis for the patient should PRIMARILY be based on:

1. prior level of function.

2. left knee range of motion.

3. use of a cervical collar.

4. upper extremity sensory integrity.

Correct answer: 1
Understanding a patient’s prior level of function is the most critical factor in a patient’s history to determine a reasonable
prognosis,
as this information establishes the baseline for recovery. Although knee range of motion and upper extremity sensory integrity
should be monitored, neither is more critical than prior level of function as a determinant of prognosis. Although a cervical collar
is a
potential intervention, its use is not more critical than prior level of function as a determinant of prognosis.

131. During examination of a patient, a physical therapist visually observes signs of Charcot disease. Which of the following
signs or symptoms would MOST likely be present and consistent with this diagnosis?

1. Erythema

2. Sharp or stabbing pain in the forefoot

3. Increased sensitivity to touch

4. Open ulcer over the second metatarsal head

Correct answer: 1
A Charcot joint is painless and usually undetected by the patient secondary to decreased sensation and neuropathy. Because of
the
increase in blood flow, patients present with erythema.

132. A physical therapist is reviewing the medical record of a patient in the intensive care unit. The patient was admitted the
previous night through the emergency department after a motorcycle accident resulting in a fractured right femur. The
therapist notes a physician’s order for a Doppler study of the left leg. The therapist should:

1. proceed with the evaluation and intervention without any changes.

2. withhold physical therapy until results of the study are obtained and interpreted by the physician.

3. proceed with the evaluation and limit intervention to transfer to a bedside chair.

29
4. obtain clearance from the nurse to provide intervention for the patient.

Correct answer: 2
A complete physical therapy evaluation and treatment is contraindicated due to a possible deep vein thrombosis. A physician’s
order
for a Doppler study indicates possible deep vein thrombosis, so the physical therapy should not be conducted until the Dopler
study
is completed and the results analyzed by the physician. Transfer from bed to chair is contraindicated due to possible deep vein
thrombosis. The nurse alone should not be providing clearance, until the Doppler study has been completed and interpreted.

133. A physical therapist is working with an outpatient who had a cerebrovascular accident and currently lives in an assisted-
living facility. Which of the following statements is the MOST appropriate functional goal for this patient?

1. The patient will be able to don an ankle-foot orthosis with assistance.

2. The patient will independently walk 165 ft (50 m) with a straight cane from the bedroom to the cafeteria.

3. The patient will have Normal (5/5) strength of the quadriceps muscles.

4. The patient’s balance will improve to be able to independently stand on the involved lower extremity for 20
seconds.

Correct answer: 2
The goal in option 2 is the only goal that is objective, measurable, and functional. The goals in options 1, 3, and 4 lack function
or
specific measurable objective information.

134. A physical therapist is examining a patient with congestive heart disease who takes diuretics. Which of the following
lower extremity conditions is MOST likely?

1. Pallor

2. Shiny skin

3. Pitting edema

4. Absence of hair growth

Correct answer: 3
When the right side of the heart starts to fail, fluid collects in the feet and lower legs, causing pitting edema. Pallor, shiny skin,
and
absence of hair growth are characteristic of arterial disease, but not associated with congestive heart failure.

135. A physical therapist is examining the integumentary system of a patient and notes an area of black, hardened eschar on
the tip of the first toe. The therapist would MOST likely conclude that the patient has which of the following conditions?

1. Frostbite

2. Diabetes

3. Only superficial damage of the skin at that site

4. At least full-thickness damage of the skin at that site

Correct answer: 4
The presence of black necrotic tissue indicates full-thickness loss of the skin. The cause of the necrosis cannot be determined
with the information in the stem.

136. A physical therapist is prescribing an exercise program to assist a patient in a weight loss program. The patient has type
1 diabetes. The patient should be advised to begin an exercise session only if the blood glucose level is:

1. below 60 mg/dL.

2. above 250 mg/dL.

3. between 60 and 100 mg/dL.

4. between 100 and 250 mg/dL.

Correct answer: 4
If a person begins exercise when blood glucose is below 100 mg/dL or above 250 mg/dL, significant exercise-related
complications
may result.

137. In planning a physical therapy program for geriatric patients, an important age-related change that should be taken into
consideration is:

30
1. the inability to learn new motor tasks.

2. decreased pain sensation.

3. decreased motivation.

4. the inability to select alternative movement strategies.

Correct answer: 2
Neurological changes that are age related include decreased pain and vibratory sensation. The other changes listed do not
normally
occur with aging in the absence of pathology.

138. A physical therapist is examining the integumentary system of a patient with dark skin pigmentation and notices a
deepening of the skin color over the left posterior aspect of the calcaneus. Which of the following findings would indicate a
Stage I pressure ulcer at that site?

1. Decreased skin temperature, compared to the surrounding tissue

2. Increased skin temperature, compared to the surrounding tissue

3. Blanching with applied pressure that returns to previous coloring within 30 seconds after the removal of pressure

4. Blanching with applied pressure that returns to previous coloring 1 hour after the removal of pressure

Correct answer: 1
The onset of a Stage I pressure ulcer is characterized by nonblanching erythema and decreased skin temperature over the site
of the
ulcer. Because blanching is difficult to ascertain in patients with dark skin, use of skin temperature for assessment of pressure
ulcers
is preferred.

139. A physical therapist is studying the effects of three treatments of ataxia. Patients were recruited from a local clinic and
were randomly assigned to one of the three treatment conditions. Functional measurements were taken at 6 months after the
start of treatment. Which of the following statistical tests is MOST appropriate for data analysis in this study?

1. Factorial analysis of variance

2. One-way analysis of variance

3. Independent sample t-test

4. Repeated-measure t-test

Correct answer: 2
In a single-factor experiment, the one-way analysis of variance is applied when three or more independent group means are
compared. The descriptor one-way indicates that the design involves one independent variable, which is the case in the present
scenario. All subjects were recruited from the same clinic and underwent the same length of treatment. The only difference
among
the three groups of subjects was the treatment group to which they were assigned.

140. A physical therapist is evaluating the cranial nerves of a child who has a medulloblastoma. The child’s right eye deviates
medially. The child has impairment of which of the following cranial nerves?

1. Oculomotor (III)

2. Trochlear (IV)

3. Abducens (VI)

4. Vagus (X)

Correct answer: 3
The oculomotor nerve (III) controls the inferiomedial eye muscles. The trochlear nerve (IV) controls inferiolateral eye
movement.
The abducens nerve (VI) controls lateral eye movement. Damage to this nerve causes the eyeball to deviate medially due to
weakness of the lateral rectus muscle. The vagus nerve (X) does not control the eye muscles.

141. A physical therapist is preparing to teach a patient with Guillain-Barré syndrome to transfer from wheelchair to mat table.
The patient weighs 150 lb (68 kg) and has Poor (2/5) strength in both of the lower extremities and Fair (4/5) strength in the
left upper extremity. The right upper extremity has Normal (5/5) strength. Which of the following assisted transfers is MOST
appropriate for the patient’s initial session?

1. Two-person lift to the right side

2. Sliding-board transfer to the right side

3. Hydraulic lift from wheelchair to mat

4. Full standing-pivot transfer to the right side

31
Correct answer: 2
Because the patient has significant weakness in both lower extremities, a standing pivot transfer would not be safe. However,
since
the patient does have fair to normal upper extremity strength and is not totally dependent, a hydraulic lift or two-person lift
would not
be appropriate because it would not enhance the patient’s functional skills. The most appropriate transfer method would be for
the
patient to use a sliding board transfer to the strongest (right) side.

142. A physical therapist has been working with a patient who has a spinal cord injury. To document that the patient has been
educated about skin care, the therapist should record that the patient:

1. is unable to tolerate more than 1 hour in a wheelchair.

2. has good strength of the scapular depressors and elbow extensors.

3. is able to state three causes of skin breakdown.

4. is able to perform 10 push-ups in the wheelchair.

Correct answer: 3
Option 3 is the only one stated clearly in educational terms. The other options are objectives based on observations.

143. Increased residual volume is LEAST likely to be a finding in pulmonary function testing of a patient with which of the
following conditions?
1. Atelectasis
2. Bronchiectasis
3. Chronic bronchitis
4. Emphysema

Correct answer: 1
Atelectasis occurs when one or more segments or lobes of the lung is collapsed. It often occurs following thoracic or upper
abdominal surgery, when the patient has a reduced total lung capacity, functional residual capacity, and residual volume.
Bronchiectasis is an abnormal dilation of the bronchi and bronchioles and results in increased residual volumes. The primary
symptom of chronic bronchitis is chronic productive cough and results in increased residual volume.

144. During pregnancy, which of the following exercises is CONTRAINDICATED?

1. Curl-ups

2. Bridging

3. Double leg lifts

4. Deep breathing with forced expiration

Correct answer: 3
During pregnancy and postpartum, the stretched abdominal muscles are unable to stabilize the lower back as the legs are
raised.
Attempting to perform double leg lifts can overwork the abdominal muscles and cause damage to spinal joints.

145. A patient who is being taught a new motor task is MOST likely to require full attention to the task and extrinsic feedback
during which phase of motor learning?

1. Cognitive

2. Associative

3. Autonomous

4. Executive

Correct answer: 1
Early learning in the 1st phase of motor learning (the cognitive phase) requires full attention to the task and extrinsic feedback.
As the task is learned, feedback can become more intrinsic and less attention to task is required.

146. A patient with a recent ankle injury reports anterior ankle pain while walking. Pain is reproduced ONLY during passive
rotation of the lower leg. The patient most likely sustatined which of the following injuries?

1. Syndesmosis sprain

2. Calcaneocuboid sprain

3. Fifth metatarsal fracture

4. Tibial stress fracture

Correct answer: 1
Option 1 tests the integrity of the tibiofibular syndesmosis. With a positive test result, the patient will have reproduction of
symptoms

32
and in this case, anterior ankle pain. Lateral (external) rotation of the tibia may potentially compress the calcaneocuboid joint
but
would not stress the ligaments. A 5th metatarsal fracture would not present with anterior ankle pain. Option 4 does not load the
tibia and/or muscles enough to provoke pain from a stress fracture.

147. To avoid the appearance of increased motion, what movement MUST be prevented during goniometric measurement of
shoulder abduction?

1. Upward rotation of the scapula

2. Medial (internal) rotation of the shoulder

3. Lateral (external) rotation of the shoulder

4. Lateral flexion of the trunk to the opposite side

Correct answer: 4
During measurement of shoulder abduction, the trunk should be kept straight. If the patient is allowed to laterally flex the trunk,
it will give the appearance of increased shoulder abduction, but the motion would be occurring in the spine and not in the
shoulder.
Upward rotation of the scapula and lateral (external) rotation of the shoulder would normally accompany the motion of shoulder
abduction. Medial (internal) rotation of the shoulder would decrease the available abduction.

148. Which of the following pulmonary function tests measures the average total amount of air moved during inspiration in a
patient with normal breathing?

1. Vital capacity

2. Inspiratory capacity

3. Tidal volume

4. Inspiratory reserve volume

Correct answer: 3
Vital capacity is measured from maximum inhalation to maximum exhalation. Inspiratory capacity measures volume with
maximal
inhalation. Tidal volume measures total volume of air moved during either inhalation or exhalation over a specific period of time
(usually 1 minute) and then divided by the ventilatory rate. Inspiratory reserve volume measures inspiratory volume beyond
normal inspiration.

149. A physical therapist is evaluating a patient who has a chest tube. The therapist accidentally knocks over the collection
reservoir. The therapist should return the reservoir to the upright position and:

1. hang it on an intravenous pole.

2. place it on the patient’s bedside table.

3. hang it from the side of the bed.

4. contact the physician.

Correct answer: 3
The reservoir must be placed below the level of the heart. Notifying the nurse would be sufficient. Notifying the physician is
unnecessary.

150. A physical therapist is examining a patient with an upper lumbar radiculitis. Which of the following combinations of
movements is MOST likely to reproduce the pain associated with the radiculitis?

1. Supine knee extension with hip flexion

2. Supine knee flexion with hip flexion

3. Prone knee flexion with hip extension

4. Prone knee extension with hip extension

Correct answer: 3
Supine knee extension with hip flexion places a stretch on the sciatic nerve, which involves lower lumbar nerve roots (L4–S3).
Knee
flexion with hip flexion would not place a stretch on the femoral nerve. An upper lumbar radiculitis involves the nerve roots that
form
the femoral nerve (L2–L4). The femoral nerve can be stretched or provoked, by extending the hip while flexing the knee. Prone
hip
extension would place a stretch on the femoral nerve, but there would be less stretch with the knee in extension than in flexion.

151. A patient with an impairment that limits hip flexion to 105° will benefit MOST from which of the following assistive
devices and modifications to accomplish activities of daily living?

33
1. Elevated chair-seat height

2. Long-handled reacher

3. Decreased rise in stair height

4. Sock aid

Correct answer: 4
Normal sitting requires about 105° of hip flexion, so an elevated chair would not be necessary. Reaching would not be as
restricted as donning socks. The reacher would not be effective in assisting with shoes and socks and may be overused. Stair
climbing requires about 60° to 70° of hip flexion, and step height modification would not be needed. To reach to the foot,
approximately 120° of hip flexion is required. If a modified procedure is chosen with the foot across the opposite thigh, at least
110° of hip flexion is still required. With only 105° of hip flexion, the patient would have most difficulty putting on socks.

152. A patient with hemiparesis demonstrates foot drop during the swing phase of gait. The MOST appropriate application of
functional electrical stimulation would be to treat both the tibialis anterior and which of the following muscles?

1. Gastrocnemius

2. Tibialis posterior

3. Extensor digitorum longus

4. Fibularis (peroneus) longus

Correct answer: 3
During the swing phase of gait, the foot is dorsiflexed to clear the floor. Muscles active during this phase of gait are the tibialis
anterior, extensor hallucis longus, and the extensor digitorum longus. The gastrocnemius, tibialis posterior, and fibularis
(peroneus)
longus are active during stance phase of gait. Therefore, of the muscles listed, the extensor digitorum longus is the best choice.
If
only the tibialis anterior was stimulated, the foot would go into inversion and dorsiflexion, which could cause the patient to land
on
the lateral side of the foot at initial contact, causing injury. Addition of the extensor digitorum longus would bring the foot up
into
more dorsiflexion and avoid excessive inversion.

153. A physical therapist plans to study the effect of cold compresses on passive range of motion in two groups of subjects: an
experimental group and a control group. For the experimental group, the cold compress will be applied to the hamstring
muscles 1 time/day for 5 days. Which of the following experimental methods is MOST appropriate for this study, in order to
compare the effect of the cold compresses between the two groups.?

1. For the experimental group, gather range-of-motion data prior to each treatment.

2. For both groups, measure range of motion on day 5.

3. For both groups, measure range of motion on days 1 and 5.

4. For the experimental group, measure range of motion every day. For the control group, measure range of motion on
days 1 and 5.

Correct answer: 3
The most appropriate design would be a two-group pretest-posttest. Both groups are measured at the same times, but only the
experimental group receives treatment. Gathering information from patient records may not provide the same accuracy as
would be
provided if the researcher had performed all of the measurements directly. Measuring ROM only on day 5 would not account for
any
pretest differences between groups. Measuring the experimental group more frequently than the control group may have an
effect
on the dependent variable.

154. A physical therapist is working with a patient who is aware of being terminally ill. What is the MOST appropriate
intervention when the patient wants to talk about the prognosis?

1. Discourage discussion of death or dying.

2. Refer the patient for pastoral counseling.

3. Relate the therapist’s experiences with other patients.

4. Encourage the patient’s expression of feelings.

Correct answer: 4
Patients should be encouraged to express their feelings. Comparisons to other patients who are dying, in an effort to assure the
patient he is not alone, takes away from this patient’s feelings. Denial of death would not be good for the patient, since he must
ultimately cope with the inevitable. Pastoral counseling would be an option, but the therapist should be ready to listen to the
patient, encourage expression of feelings, and avoid denial.

155. A physical therapist is examining a patient who was unaware of a circular ulcer on the plantar surface of the second
metatarsal head. The patient is noted to have hammer toes. Which
34 additional findings are MOST likely?
1. Normal arterial pulses

2. Increased sweating over the foot

3. Increased pain with elevation of the foot

4. Altered biomechanics when weight-bearing

Correct answer: 4
The location of the painless ulcer is characteristic of a neuropathic (diabetic) ulcer. Loss of sensation renders the patient
unaware
of the repetitive trauma from altered biomechanics due to weakness of the intrinsics, which leads to hammer toes, and abnormal
pressure on the plantar foot when weight-bearing. Increased pain with foot elevation is characteristic of arterial ulcers, which
are painful. Normal arterial pulses may be present with a venous ulcer, but arteries are often calcified with advanced diabetes.
A neuropathic foot is warm and dry.

156. A physical therapist examining wrist-joint play finds restriction in the direction indicated by the arrow. To address the
restriction, the therapist should include an intervention to increase which motion of the index finger (2nd digit)?

1. Flexion

2. Extension

3. Abduction

4. Rotation

Correct answer: 1
The therapist is shown performing a volar glide, which is the same joint motion used for finger flexion. Limited motion in this
direction indicates limited ability to perform finger flexion.

157. A physical therapist is treating a patient who has diabetes mellitus. Which of the following information is MOST important
to include when educating the patient about the benefits of exercise?

1. Blood glucose levels increase during exercise.

2. Weight training is preferred to aerobic exercise for patients with diabetes.

3. Exercise can reduce the amount of insulin the patient must administer.

4. Exercising before meals is recommended for patients with diabetes.

Correct answer: 3
One of the benefits of exercise is improved insulin sensitivity, which reduces the amount of insulin needed. Blood glucose levels
decrease during exercise. Patients with diabetes should not exercise before meals because this timing increases the risk of
hypoglycemia. Aerobic exercise is just as important for people who have diabetes as for healthy people, so weight training is not
preferable.

158. A patient has pain, edema, and tenderness over the medial border of the hand. The patient also shows changes in the
color and temperature of the skin, hyperhidrosis, and progressive joint stiffness in the wrist and hand. The MOST likely cause
of the patient’s signs and symptoms is:

1. cervical disc disease.

2. Raynaud phenomenon.

3. complex regional pain syndrome.

4. carpal tunnel syndrome.

Correct answer: 3
All of these symptoms are indicative of complex regional pain syndrome. Cervical disc disease does not produce swelling in
the hand, color and temperature changes, or hyperhidrosis. Raynaud phenomenon results in pain, pallor, and coolness,
but no hyperhidrosis.
Although carpal tunnel syndrome can also show sympathetic nervous system abnormalities, compression of the median
nerve
would refer symptoms to the lateral border of the hand (thumb area). With carpal tunnel syndrome, the patient may also
display thenar muscle weakness.

159. A physical therapist is planning a patient education program for a group of patients with chronic low back pain. The
therapist should use which of the following strategies to MOST increase the likelihood that the patients will utilize the proper
body mechanics they learn?

1. Ask the patients to demonstrate use of proper body mechanics.

2. Provide a reference list of articles describing body mechanics.

3. Ask the patients to describe actions they do that increase their back pain.

4. Provide information on the frequency of low back injuries due to improper body mechanics.

35
Correct answer: 1
Evaluation of the patients’ retention of the information presented in the program can be enhanced by asking the patients
questions
about the program information, having the patients ask questions about the program, having the patients demonstrate what
they
have learned, and testing the patients about the program material. Option 1 is the only option that includes one of these
techniques.
The other options do not require active participation by the patients and do not address specific interventions.

160. A physical therapist should expect a patient to describe pain associated with a 5-year history of rheumatoid arthritis as:

1. morning pain with stiffness that improves with activity.

2. pain that is worse at the beginning of an activity.

3. constant pain that lasts throughout the day.

4. pain that increases throughout the day.

Correct answer: 1
Chronic inflammatory disorders such as rheumatoid arthritis are often associated with morning pain and stiffness that decrease
throughout the day with activity. Pain that is worse at the beginning of an activity implies acute inflammation. Constant pain
suggests
tumors or a visceral cause. Pain that increases throughout the day indicates increased congestion in a joint.

161. Use of ice massage to affected area(s) is MOST appropriate for a patient who has which of the following problems?

1. Trigger point in the upper trapezius muscle

2. Spasticity of the plantar flexor muscles after stroke

3. Exercise-induced soreness of abdominal muscles

4. Decreased extensibility of bilateral hamstrings muscles

Correct answer: 1
Ice massage is used to treat a small area, such as a trigger point in a muscle. Ice is applied to produce analgesia and allow for
deep
pressure massage over the trigger point and stretching of the muscle. Treating large areas, such as the lower leg (as in option
2),
is done more efficiently with cold baths or cold packs than with ice massage. Cryotherapy increases stiffness and decreases
tissue extensibility (p 76) and so would not be indicated for stretching hamstrings.

162. A patient presents with sudden onset of weakness of the facial muscles on the right side. The patient is unable to wrinkle
the forehead, smile, pucker the lips, or wrinkle the nose. There is an absence of tearing in the patient’s right eye, diminished
taste sensation on the right side of the tongue, and dryness of the mouth. The patient’s corneal reflex is absent on the right
but normal on the left, and pinprick and temperature sensation are normal on both sides of the face. This presentation is
characteristic of:

1. trigeminal neuralgia.

2. Bell palsy.

3. left cortical cerebrovascular accident.

4. oculomotor nerve damage.

Correct answer: 2
Trigeminal neuralgia causes facial pain. All of the signs and symptoms in the question are indicative of a partial nerve lesion
affecting
the facial nerve. This type of lesion is Bell palsy. A left cerebrovascular accident affects only the lower facial muscles below the
eye.
Damage to the oculomotor nerve causes paralysis of one or more of the muscles that moves the eyeball.

163. Which of the following interventions is MOST appropriate for a patient with juvenile rheumatoid arthritis who is
experiencing painful swelling of both knees?

1. Resistive exercises

2. Stretching to prevent contractures

3. Gentle, active exercises

4. Walking program

Correct answer: 3
All of the options, except for gentle, active exercises, are precautions or contraindications for this patient.

164. A physical therapist is considering whether the results of a study can be usefully applied to practice settings other than
the one in which the study was conducted. Which of the following36 types of validity is of MOST concern in this scenario?
1. Internal

2. Construct

3. Concurrent

4. External

Correct answer: 4
External validity is concerned with the issue of how the results of the research can be generalized.

165. Treatment of a patient with hemophilia who has a subacute hemarthrosis of the knee should INITIALLY include:

1. active assistive range-of-motion exercise to the knee.

2. instruction of the patient for weight bearing to tolerance.

3. gentle resistive range-of-motion exercise to the knee.

4. continuous immobilization of the knee in an extension splint.

Correct answer: 1
In this stage of hemarthrosis, there is still some bleeding into the joint space, but it is not as extensive as during the acute
phase.
Therefore the patient will benefit from range-of-motion exercise to prevent contracture. The patient may need active-assist, as
there
may still be pain or edema in the joint that prevents independent performance of range of motion. The mechanical trauma of
weight
bearing to tolerance at this stage may impinge on and damage the pathologic synovium within the joint. Resistive range of
motion is
more appropriate when pain and swelling have subsided and no bleeding is occurring. Continuous immobilization in the extended
position will promote contracture in the edematous knee.

166. A patient with a traumatic brain injury is being discharged to home after completion of inpatient rehabilitation. Which of
the following assessment tools should be used to BEST assess the patient’s potential?

1. Glasgow Outcome Scale

2. Fugl-Meyer Assessment

3. Rancho Los Amigos Levels of Cognitive Functioning Scale

4. Sickness Impact Profile

Correct answer: 1
The Glasgow Outcome Scale is used as a general outcome measure, and in prognostic studies. The Fugl-Meyer Assessment
should
be used only with patients who have had a stroke. The Rancho Los Amigos Levels of Cognitive Functioning is a descriptive scale
that
outlines a predictable sequence of cognitive and behavioral recovery as a patient emerges from a coma. The Sickness Impact
Profile
is used to measure general health status and is self-administered; therefore, it would not be useful as a prognostic discharge
tool for
a patient with a traumatic brain injury.

167. Which of the following examination findings would MOST likely be positive in a patient with cauda equina syndrome?

1. Ankle clonus

2. Babinski sign

3. Urinary retention

4. Positive prone knee flexion test

Correct answer: 3
Cauda equina causes lower motor neuron dysfunction, including urinary retention and incontinence. Positive Babinski and clonus
responses indicate upper motor neuron dysfunction. A positive prone knee flexion test is more likely with upper lumbar nerve
root impairment.

168. A patient with low back pain has been undergoing treatment for two sessions. During the current treatment session, the
patient tells the physical therapist that the pain is centralizing with the extension exercises but is as intense as it was at the
first treatment session. The patient is frustrated by this reaction. The therapist's MOST appropriate response is to:

1. continue with the present program.

2. eliminate the extension exercises.

3. consult the patient’s physician about the situation.

37
4. progress to trunk flexion exercises.

Correct answer: 1
With an extension exercise program, centralization indicates that the patient is improving, even though the pain may be just as
intense as it was. Even though the patient may feel frustrated, he is actually improving; and therefore the therapist should
continue
with the extension exercise program. There is no need to call the physician at this point, since the centralization is perfectly
normal.
Although trunk flexion may be used later in the intervention, it would not be indicated at this point because extension is
working.

169. A patient is receiving physical therapy intervention for rheumatoid arthritis, which is in remission. Which of the following
interventions is MOST appropriate for the patient?

1. Contract-relax stretching for tight structures

2. End-range mobilization techniques

3. Elimination of functional activities of involved parts

4. Strengthening exercises for weak muscles

Correct answer: 4
Contract-relax stretching for tight structures is not recommended for soft tissues compromised by rheumatoid arthritis. Grade IV
mobilization techniques are contraindicated for soft tissues compromised by rheumatoid arthritis. The elimination of functional
activities is not required. Strengthening exercises are the most appropriate intervention for a patient with rheumatoid arthritis in
remission.

170. A patient with chest pain from myocardial ischemia will MOST likely exhibit:

1. increased pain upon chest-wall palpation.

2. increased pain with deep breathing.

3. relief with nitroglycerin (Nitrostat) ingestion.

4. relief with antacid ingestion.

Correct answer: 3
Increased pain with chest-wall palpation is more indicative of a musculoskeletal origin of pain. Increased pain with deep
breathing is
more indicative of a pulmonary origin of pain. Nitroglycerin (Nitrostat) is a common vasodilator that is prescribed for patients
who
have angina. Ingestion of a vasodilator will improve myocardial blood flow and help relieve ischemia and its manifestations.
Relief of
pain with antacid ingestion is more indicative of referred pain from peptic ulcer disease.

171. During a posture examination, the physical therapist notes that both of the patient’s patellae point inward when viewed
from the front of the patient. The MOST likely cause of this problem is excessive:

1. femoral anteversion.

2. weakness of the vastus medialis.

3. genu varum.

4. medial tibial torsion.

Correct answer: 1
The most common cause of inwardly pointing or “squinting patellae” is excessive femoral anteversion. Although there is normally
8° to 15° of femoral anteversion, an excessive amount leads to squinting patellae and toeing in. The other options would all
have
a tendency to cause the patellae to point outward during standing.

172. Which of the following statements does NOT document patient outcome?

1. The patient propelled his wheelchair independently after 4 weeks.

2. The patient demonstrated independent performance of a home program after 2 weeks.

3. The patient walked 100 ft (30.5 m) with minimal assistance after 1 week.

4. The patient attended physical therapy sessions 3 times/week for 2 weeks.

Correct answer: 4
An outcome measure documents a desired performance or change in the patient’s condition over time. This may include a
description
of the patient’s function before, during, and after intervention. Option 4 does not indicate a change or function but is focused on
attendance.

173. A patient sustained a gunshot wound to the spine in the area of L1. The patient has weakness of the left lower extremity
38
1. Complete severance of the spinal cord

2. Injury to the left anterior horn of the spinal cord

3. Injury to the left side of the spinal cord

4. Injury to the central area of the spinal cord

Correct answer: 3
Complete severance of the spinal cord causes motor and sensory loss on both sides of the body. A lesion of the anterior horn
cells
causes a lower motor neuron problem and hyporeflexia. The signs and symptoms presented in the question point to a
hemisection
of the spinal cord on the left side, which gives rise to ipsilateral motor and proprioception loss and contralateral loss of pain and
temperature. Also present are hyperreflexia and a positive Babinski sign on the left side. A lesion of the central spinal cord
usually
spares the motor tracts.

174. Which of the following interventions would MOST specifically address expected complications from radiation therapy to
the pelvic region?

1. Anterior hip stretching

2. Splinting while coughing

3. Progressive abdominal strengthening

4. Pelvic stabilization exercises

Correct answer: 1
Radiation of the pelvic cavity often causes dense pelvic adhesions that can restrict motion due to pain. Of the options, stretching
exercises are most specific to address this complication.

175. Which of the following types of patients should have the most detailed and lengthy home exercise program?

1. A patient with acute symptoms

2. A patient with irritable symptoms

3. A patient who has good body awareness

4. A patient who has physical therapy visits once a week

Correct answer: 4
As the frequency of visits decreases and the time between visits increases, the patient requires a more detailed home program
with
specific progression instructions.

176. A physical therapist measures the forced expiratory volume in 1 second (FEV1) of a patient with mild pulmonary
obstructive disease and determines a FEV1/FVC (forced vital capacity) ratio of 60%. The therapist should recognize that the
patient:

1. has a normal FEV1 value.

2. will require careful monitoring during aerobic exercise.

3. should not exercise beyond 60% of the patient’s predicted maximal heart rate.

4. should not perform aerobic exercise without physician approval.

Correct answer: 2
Regarding option 1, a ratio of 75% is considered normal. Regarding option 2, the patient has a mild disease. Advanced
obstructive
disease can have values of 25%. Nonetheless, the patient has mild morbidity, so special precautions should be used. Regarding
option 3, the patient has a mild disease and is not yet showing signs of morbidity, so special precautions should not be
necessary.
There would be no need to limit heart rate to 60% if the patient tolerates the exercise. An FEV1/forced vital capacity ratio of
60% is a
mild loss. Avoidance of aerobic exercise should not be required, and special physician approval related to a value of 65% should
not
be necessary.

177. Which of the following skin color changes is MOST likely to be sign of liver disease?

1. Pallor

2. Jaundice

3. Cyanosis

39
4. Hyperpigmentation

178. Acquired immunodeficiency syndrome is MOST likely present in a patient with which of the following hematologic
conditions?

1. CD4 count of 150 cells/mL

2. High antinucleoprotein antibody titer

3. High human leukocyte antigen titer

4. Neutrophil count of less than 5000 cells/mm3

Correct answer: 1
Patients with AIDS include those who have HIV-1 and a CD4 count below 200 cells/mL. A low neutrophil count or a high ANA or
HLA
titer are not specific to the presence of HIV.

179. A patient with leukemia has developed thrombocytopenia after a bone-marrow transplant. Which of the following
measures is indicative of the status of the thrombocytopenia?

1. T4 lymphocyte count

2. Red blood cell count

3. Platelet count

4. White blood cell count

Correct answer: 3
Thrombocytopenia is an acute or chronic decrease in the number of platelets in the circulation. The T4 lymphocyte count is used
to
assess immune status in patients with HIV or AIDS. The red blood cell count is utilized to assess for presence of anemia, and the
white blood cell count would be examined to determine presence of infection or degree of immunosuppression.

180. Which of the following factors is considered to be a primary risk factor for atherosclerosis?

1. Stress

2. Obesity

3. Cigarette smoking

4. Sedentary lifestyle

Correct answer: 3
High blood pressure, cigarette smoking, and hyperlipidemia are direct or primary risk factors for atherosclerosis. Secondary risk
factors include age, gender, race, obesity, stress, and activity level.

181. Which of the following statements is an appropriately written short-term goal?

1. In 2 weeks, the patient will increase walking distance from 50 ft to 100 ft (15 m to 30 m) with a wheeled walker.

2. The patient will walk 100 ft (30 m) with a wheeled walker and minimal assistance in 3 weeks.

3. In 4 weeks, the patient will walk with minimal assistance 100 ft (30 m) without loss of balance.

4. The patient will increase walking distance from 50 ft to 100 ft (15 m to 30 m) with a wheeled walker independently
by discharge.

Correct answer: 2
Option 1 does not specify the amount of assistance. Option 2 is a measurable, specific, and objective goal. Option 3 does not
specify
the device the patient will be using. Option 4 is a goal that is to be achieved by discharge, which would be a long-term goal, not
a
short-term goal.

182. Which of the following nerves innervates the pelvic floor muscles?

1. Sciatic

2. Pudendal

3. Inferior gluteal

4. Genitofemoral

40
The pelvic floor muscles are innervated by sacral nerve roots S2–S4 through the pudendal nerve. The inferior gluteal nerve is a
motor
nerve that innervates the gluteus maximus. The sciatic nerve contains nerve roots from L4–S3 and innervates multiple muscles in
the
lower leg. The genitofemoral nerve, which is from L1–L2, is a cutaneous nerve that is sensory and not motor.

183. A patient with a recent onset of rheumatoid arthritis is MOST likely to report which of the following symptoms?

1. Heat intolerance

2. Malaise and fatigue

3. Tension headaches

4. Nausea and diarrhea

Correct answer: 2
Malaise and fatigue are common symptoms during the beginning stages of rheumatoid arthritis. Tension headaches, heat
intolerance,
and nausea and diarrhea are not associated with rheumatoid arthritis.

184. For a patient with type 1 diabetes who is completing a cardiovascular fitness program, what change in diabetic
management is MOST likely to be instituted as fitness increases?

1. Switching to oral rather than injected medication

2. Decreasing caloric intake for 2 to 3 hours following exercise sessions

3. Decreasing the amount of insulin taken daily

4. Increasing the amount of insulin taken daily

Correct answer: 3
Exercise has been shown to increase sensitivity of the insulin receptors, leading to a decrease in the amount of insulin required.
Administration of medication is dictated by tolerance and efficacy of the medication and would not necessarily be altered by
exercise.
In patients with moderate hyperglycemia, exercise can lead to hypoglycemia for periods of 24 to 48 hours after exercise;
therefore,
an increase in caloric intake, particularly intake of carbohydrates, would be essential.

185. A physical therapist is evaluating a patient who had a cerebrovascular accident and is exhibiting poor foot and ankle
control. When asked to lift the foot more during the midswing phase of gait, the patient dorsiflexes the ankle with excessive
eversion. To correct this problem, facilitory electromyographic biofeedback is MOST likely to be used with which of the
following muscles?

1. Extensor digitorum longus

2. Tibialis anterior

3. Fibularis (peroneus) brevis

4. Flexor hallucis longus

Correct answer: 2
The patient has too much eversion during the midswing phase. This is probably caused by too much activity in the extensor
digitorum
longus or extensor hallucis longus. To correct the situation, the tibialis anterior should be facilitated, because it dorsiflexes and
inverts
but doesn’t cause eversion. The fibularis brevis, an everter, would cause plantar flexion of the foot rather than dorsiflexion. The
flexor
hallucis longus does not dorsiflex the ankle.

186. What skin change associated with aging has the GREATEST effect on wound healing?

1. Reduction in sensation

2. Decreased elasticity of the skin

3. Decreased epidermal proliferation

4. Change in pigmentation

Correct answer: 3
Wounds heal via a complex process involving re-epithelialization. With advanced aging, the rate of epidermal proliferation
decreases.

187. Which of the following questions is MOST appropriate for a functional outcome measurement tool?

1. How high can you lift your arm?

2. How long have you had this pain?

41
3. How often do you walk with a limp?

4. How much does this leg bother you?

Correct answer: 3
Option 1 does not ask about function. It is an impairment-related question, not a function-related question. Options 2 and 4 do
not
ask about function, and even with improvement, neither would relate to function. Option 3 is taken from the Short
Musculoskeletal
Function Assessment. It addresses function and can be used as an outcome question.


18
A patient has adhesive capsulitis of the shoulder joint. The range-of-motion examination reveals restricted lateral
8.
(external) rotation and abduction of the shoulder. Which of the following mobilization procedures should be performed for
the patient FIRST?

1.
Posterior glide

2.
Distraction

3.
Anterior glide

4.
Lateral (external) rotation

Correct answer: 2
For this patient, the first mobilization procedure would be distraction of the glenohumeral joint. The distraction separates the
joint
surfaces and is used as a test of joint play. The distraction can also help increase joint play. Distraction may also be used in
conjunction with the other mobilization techniques listed. Later mobilization techniques would most likely include anterior glide.

189. A patient with right piriformis syndrome is referred to physical therapy for evaluation and intervention. The patient’s
history includes a total hip arthroplasty on the right side 2 years ago. Which of the following interventions requires added
precaution for this patient?

1. Transcutaneous electrical nerve stimulation

2. Continuous ultrasound

3. Hot packs

4. Massage to the right hip

Correct answer: 2
The only one of the interventions listed that requires precaution because of the total hip replacement is continuous ultrasound.
However, that does not mean that ultrasound is contraindicated for this patient. Transcutaneous electrical nerve stimulation may
be used over metal implants. Hot packs and massage would not affect the total hip prosthesis.

190. An adult patient who was involved in a motor vehicle accident has sustained multiple traumas, including fractured ribs on
the right side. The patient is unconscious, intubated, and receiving mechanical ventilation in the intensive care unit. Chest
radiographs show the development of an infiltrate in the right lower lobe during the past 2 days. Crackles and wheezing are
heard over the right lower lung fields. Which of the following chest physical therapy programs is MOST appropriate?

1. Manual hyperventilation and suctioning while positioned on the left side

2. Positioning in supine for suctioning, followed by manual hyperventilation while positioned on the left side

3. Suctioning, percussion, and vibration while positioned on the right side

4. Positioning on the left side for deep breathing exercises only

Correct answer: 1
In order to optimally clear the congestion that has developed in the right lower lobe, the patient should be positioned on the left
side
to allow gravity to help drain the secretions to the proximal airways. This patient is also unconscious and intubated, so in
addition to drainage, manual hyperinflation is necessary to provide increased ventilation to help to mobilize secretions, then
suctioning to clear
the secretions. Suctioning first then hyperventilating while in left-sidelying is less effective than the order suggested in Option 1.
Positioning on the right side would not drain the right lower lobe, and since the patient is lying on the right side, the only place
to
percuss and vibrate would be the left side, which has no pathology. Finally positioning on the left side with deep breathing
exercises
will not be effective in mobilizing secretions, especially since the patient is unconscious and unable to actively perform deep
breathing exercises.

191. A physical therapy student’s learning style is described as a preference for active experimentation and concrete
experience. For mastering evaluation of musculoskeletal dysfunction, which of the following educational strategies would be
the LEAST effective approach for the supervising physical therapist to use with this student?
42
1.
Practicing and providing feedback on examination skills

2.
Role-playing the interview process

3.
Assigning readings about the dysfunction

4.
Self-pacing performance of the evaluation

Correct answer: 3
Practice and feedback of examination skills, role playing the interview process, and self-paced performance involve active
learning
with other people in everyday situations. Active learning relies on feeling and intuition rather than logic and reasoning. Readings
do not meet the learning needs of active experimentation or concrete experience.

192. A physical therapist is planning a full-immersion hydrotherapy treatment. This treatment is CONTRAINDICATED for
which of the following conditions?

1. Osteoporosis

2. Peripheral edema

3. Exercise-induced asthma

4. Uncontrolled hypertension

Correct answer: 4
Due to the circulatory adaptations needed, patients with uncontrolled hypertension would not be permitted to use full-immersion
techniques. Full-immersion hydrotherapy results in improvement of edema due to the effects of water pressure, reduced
severity
of exercise-induced asthma in comparison to land-based activities, and reduced stress on skeletal areas in osteoporosis.

193. During examination of a patient, a physical therapist notes bradycardia. The patient reports constipation and cold
intolerance. Which of the following conditions is the MOST likely cause of this sign and symptoms?

1. Hypothyroidism

2. Hyperthyroidism

3. Hyperparathyroidsim

4. Hypoparathyroidsim

Correct answer: 1
Hypothyroidism would most likely cause the signs and symptoms. Hyperthyroidism causes diarrhea, rather than constipation.
Hyperparathyroidism is not associated with bradycardia. Hypoparathyroidism is associated with arrhythmias and either
constipation or diarrhea.
194. A patient who had a C6 traumatic spinal cord injury 1 month ago reports a severe, pounding headache and restlessness
while standing in a standing frame. The patient’s vital signs indicate hypertension and bradycardia. The MOST appropriate
immediate response of the physical therapist is to:

1. instruct the patient in deep breathing techniques.

2. apply a cervical cold pack.

3. remove the abdominal gait belt.

4. assess for catheter blockage.

Correct answer: 4
Neither deep breathing techniques nor a cervical cold pack would be an effective response to this emergency situation. Although the
abdominal
gait belt might be a contributing factor, it is not the most likely primary cause. This patient scenario is indicative of
autonomic dysreflexia, a pathological reflex. The most common cause is bladder distention or urinary retention, which is likely to
be caused by a urinary catheter blockage.

195. Which of the following lower extremity proprioceptive neuromuscular facilitation patterns is MOST appropriate for a
patient who needs strengthening of the tibialis posterior?

1. Hip extension, abduction, and medial (internal) rotation, with ankle plantar flexion and eversion

2. Hip flexion, adduction, and lateral (external) rotation, with ankle dorsiflexion and inversion

3. Hip extension, adduction, and lateral (external) rotation, with ankle plantar flexion and inversion

4. Hip flexion, abduction, and medial (internal) rotation, with ankle dorsiflexion and eversion

Correct answer: 3
The tibialis posterior plantar flexes and inverts the foot. This pattern requires the specific action of that muscle. The other
43
196. A physical therapist is evaluating a patient with low back pain and associated symptoms. Which of the following findings
should cause the therapist to refer the patient back to the physician?

1. Good (4/5) strength of the hamstrings

2. Pain radiating down the back of the thigh into the calf

3. Positive straight-leg test at 60° of hip flexion

4. Saddle anesthesia around the perineum

Correct answer: 4
Options 1, 2, and 3 are typical with low back pain, and none alone requires immediate referral. Saddle anesthesia around
the
perineum is a sign of cauda equine syndrome and usually requires immediate treatment by someone other than a physical
therapist.

197. Which of the following afferent fiber types transmits information regarding aching pain?

1. Large unmyelinated fibers

2. Small unmyelinated fibers

3. Small myelinated fibers

4. Large myelinated fibers

Correct answer: 2
Aching pain is transmitted on small, unmyelinated Type C fibers.

198. A therapist is measuring passive knee range of motion in a patient. The measurements obtained are shown in
photographs A and B. The MOST likely cause of the difference in knee range of motion is:

1. restriction in the knee joint capsule.

2. tightness in the rectus femoris.

3. weakness of the hamstrings.

4. tightness in the vastus medialis.

Correct answer: 2
Capsular restriction would show up in both measurements. In photograph A, there is more knee flexion present with the hip flexed.
In this position the rectus femoris is on slack across the hip joint, allowing greater range of knee flexion. In photograph B the rectus
femoris is stretched over both the knee joint and the hip joint, so tightness in the rectus femoris would restrict knee flexion.
Photograph B also shows hip joint flexion. Hamstring weakness would not affect passive range of motion. Vastus medialis tightness
would affect both measurements.

199. Which of the following monofilament grades indicates that only protective sensation is intact?

1. 3.61

2. 4.37

3. 5.07

4. 6.65

Correct answer: 3

The 5.07 monofilament supplies the least amount of force that can be sensed by patients with only protective sensation intact. Ability
to sense only the 6.65 monofilament indicates a loss of protective sensation. Ability to sense the 3.61 and 4.37 monofilaments
indicates fully normal sensation.

200. Which of the following changes is a result of stimulation of the sympathetic nervous system?

1. Increased blood flow to skin

2. Decreased blood glucose

3. Dilation of bronchioles

4. Rise in heart rate

The sympathetic nervous system is responsible for adjustments in preparation for emergency situations (fight of flight). With
stimulation
of the sympathetic nervous system, the heart rate would rise. Blood flow would be shunted from, not to, the skin. Blood glucose
levels would increase,not decrease, in preparation for increased work. Bronchioles would constrict, not dilate.

44

You might also like